Вы находитесь на странице: 1из 75

CD- COMPRE EXAM 1

 
 
 
1. A nursing instructor asks a nursing student to describe live or attenuated vaccines.  The
student tells the instructor that these types of vaccines are:
a. vaccines that have their virulence (potency) diminished so as to not produce a full-
blown clinical illness.
b.  vaccines that contain pathogens made inactive by either chemical or heat.
c.  bacterial toxins that have been made inactive by either chemicals or heat.
d.  vaccines that have been obtained from the pooled blood of many people and provide
antibodies to a variety of diseases.
 
    2.  Passive immunization is given as a preventive measure when
a. Routine immunization is given
b. Clinical recovery is on the way
c. Exposure to a frank case has been observed
d. Symptoms of the disease is manifested by the patient
 
   3.  A 6-month-old infant receives a DPT immunization at a well-baby clinic. The mother returns
        home and calls the clinic to report that the infant has developed swelling & redness at the   
        site of injection. The nurse tells the mother to
a.       apply a warm compress to the injection site
b.       bring the infant back to the clinic
c.       apply an ice pack to the injection site
d.       monitor the infant for a fever
   4.  A nursing student is assigned to administer immunizations to children in a clinic. The
        nursing instructor asks the student about the contraindications to receiving an immunization.
        The student responds correctly by telling the instructor that a contraindication to receiving  
         an immunization is if a child has
                        a. a cold                                                            c. otitis media
                        b. mild diarrhea                                                 d. a severe febrile illness
  5. A 12-month-old received immunizations at his well-child visit. Fourteen days later, the
      mother reports that the child has red maculopapular rash on the trunk & extremities. This rash
      is most likely due to which of the following vaccines?
            a. DPT
            b. OPV
            c. MMR
            d. Hib
6. When a woman receives rubella vaccination, a nurse would give her which of the following
instructions?
            a. “Don’t get pregnant for at least 3 months.”
            b. “Refrain from eating eggs & egg products for 48 hours.”
            c. “Limit contact with visitors for at least one week.”
            d. “Avoid breast-feeding the baby for at least 24 hours.”
7. A 12 years old child comes into the clinic for the MMR vaccines for entry into the 6 th grade the nurse
asks the mother if the child had changes in health since last seen. Which statement by the mother would
indicate a potential contraindication to MMR immunization?
a. “ My child had neomycin sulfate for acne and develop hives and difficulty in breathing”
b. After the last MMR my child developed a low grade and had a rash  for the couple of days 
c. My child’s brother is taking corticosteroids for juvenile rheumatoid arthritis
d. My child just had a tuberculin TB test.
8. The parents of an 18-month-old baby who developed signs of tetanus are concerned about how the
disease will affect their child’s intellectual ability in the future. The nurse’s best response will be
a. “The child’s intellectual functioning may be damaged.”
b. “The child’s intellectual functioning should remain intact.”
c. “The child’s intellectual functioning may be temporarily retarded.”
d.  “The child’s intellectual functioning depends on the severity of complications.”
9. A 7 day old child is brought to the health center because of inability to suck since yesterday. You
suspect that this is a case of neonatal tetanus. As you asses the child which of the questions will you ask
the mother in order to confirm your suspision?
            a. Did the child normally suck and cry 2-4 days before she was brought to the center ?
            b. Did the child have convulsion?
            c. Have you observed stiffness of the child’s body
            d. all of these questions
10. Which of these statements about tetanus is wrong?
            a. immunity last for 5 months in infants born to immune mother
b. Immunity can be obtained after two primary doses of toxoid in mothers one moth before
delivery.
            c. recovery from tetanus does not result in solid immunity
            d. tetanus bacillus is directly transmitted from man to man.        
11. Prevention of tetanus neonatorum can be achieved through:
a. increasing the immunization coverage of women of child-bearing age with tetanus toxoid.
            b. immunizing newborns
            c. increasing the proportions of deliveries attended by the traditional birth attendants
            d. all of the above
12. If you were to conduct on the seminar on the prevention and control of tetanus neonatorium for your
new midwives, which of the following should be given emphasis?
            a. Methods equipments and techniques of asepsis in childbirth
            b. Education of mothers on the practice of asepsis in the care of umbilical stamp.
            c. Education of birth attendants on the practice of strict
            d. All of these measures
Situation:  A 7 year old boy was admitted in the hospital because of convulsion 6 hours  ago.  A week ago
the child experienced on and off cough and low grade fever.  On examination, the nurse observes nuchal
rigidity.
13. The most common cause of bacterial meningitis in children is
a. Haemopilus influenzae B                                    c. Neisseria meningitides
b. Streptococcus pnemoniae                                   d. Meningococcus
14. To confirm the diagnosis, the doctor performed lumbar puncture.  The result that would indicate that
the cause of the disease is more of bacterial in origin would be
a. Increased pressure, glucose, and protein
b. Increased pressure, protein and decreased glucose
c. Increased protein, low glucose, and pressure
d. Cloudy CSF, increased protein and glucose
e.       A mother brings the child to the health care clinic. 
15. The child has been complaining of severe headaches and has been vomiting.  The child
has a high fever, and the nurse notes the presence of nuchal rigidity in the child.  The nurse
suspects a possible diagnosis of bacterial meningitis. The nurse continues to assess the child
for the presence of Kernig’s sign.  Which finding would indicate the presence of this sign.
a. Inaability of the child to extend the legs fully when lying  supine
b. Flexion of the hips when the neck is flexed from a lying position
c. Pain when the chin is pulled down to the chest
d. Calf pain when the foot is dorsiflexed
 
16. A patient who is diagnosed with encephalitis is being treated with mannitol. Which of the following
patient outcomes should indicate to a nurse that treatment with mannitol has been effective for a patient
who has increased intracranial pressure?
            a. Decreased level of consciousness
            b. Increased urinary output
            c. Elevated body temperature
            d. Slowed papillary response
 
17. Maria Victoria, a 26-year-old female equestrian, developed fever, chills & body malaise for 5 days.
On the 7th day of illness, she developed convulsions & was rushed to a tertiary hospital. As an ER nurse in
that hospital, you noticed that the patient’s upper & lower extremities are in a flexed position. You very
well know that this is
a. decorticate rigidity                                                c. nuchal rigidity
b. decerebrate rigidity                                               d. opisthotonus
Situation: Mrs. Ferrer delivered a healthy baby girl weighing 6.8 lb. Due to knowledge deficit, the mother
was not able to bring the neonate for immunization. When the child reached 3 years old, she developed
pain on both knees & discomfort during standing and walking prompting her to prefer to sit or lie down.
The child started having fever on the 3 rd day after the leg pain. When the child was brought to a hospital, a
diagnosis of Poliomyelitis was made.
18. During the early symptoms of poliomyelitis, the client usually suffers from
            a. coryza
            b. headache
            c. seizures
            d. vomiting
Situation : Cristina, an 8-year-old girl was never given any kind of immunization because the family had
never visited a health care unit.  Now she is diagnosed to have acute poliomyelitis.
19. The major route of transmission of polio virus in poor sanitation environment is:
      a. Sewage disposal                                           
b. Water system                                                  
            c. Fecal- oral
            d.  Food stuff
20. When the virus is ingested, where does it multiply?
      a. Gastro- intestinal tract                                               
      b. Integumentary system           
      c. Urinary tract
      d. Reproductive tract
21. As a rural health nurse, your most important function in polio prevention is to:
a. Educate the public on the advantages of immunization
b. Distribute leaflets about the causes of polio
c. Call all mother for  lecture
d. Refer all children to the doctor
22. Your health teaching to Cristina, the polio victim includes:
a. Proper waste disposal                                   
b. Proper eating habits  
c. Regular health check-up
d. Advocacy in health
23. The best nursing intervention in the case of Cristina with paralysis of lower extremity due to
poliomyelitis is:
      a. Letting Cristina attend mother class   
      b. Rendering physical comfort
      c. Rendering physical therapy exercises to limit atrophy
      d. Diet instruction
24. Sight & sound of water as well as drip of the faucet disturb a patient with rabies. Which of the
following should the nurse do first in taking care of a patient with rabies?
            a. Place patient near the window
            b. Cover IV bottle with brown paper bag
            c. Give tepid sponge bath
            d. Place the patient near a utility room
25. If a dog bites a person, it should be
            a. Leashed immediately & observed for 10 days
            b. Killed immediately
            c. Brought to a veterinarian for examination to determine the presence of rabies in the
             blood
            d. punished by not giving food & water
26. A 10-year-old boy arrives with his mother at the emergency department after being bitten by a stray
dog. There is a bleeding soft tissue injury on the inner aspect of the left forearm. The first nursing action
would be
            a. Notify the barangay tanod to capture the dog
            b. Ask the mother if her son is allergic to horse serum
            c. assess the injury, vital signs, and past health history
            d. inoculate the child with human rabies immune globulin
 
27. Emilie Lopamia, a 35-year-old female was bitten by a stray dog on the left hand. She sought an advice
of the nurse in the health center. The best response of the nurse is
a. “You should observe the dog for changes in behavior so you will know if the dog is
possibly rabid or not.”
b. “You should receive immunization for rabies immediately.”
c. “You don’t have to worry because it’s just a minor wound.”
d. “You should receive immunization for rabies & tetanus as soon as possible.”
28. Emilie asked the nurse regarding the incubation period of rabies. Which of the following statements
made by the nurse is appropriate?
a. “The incubation period is about 2 weeks.”
b. “The incubation period varies depending on the site of the bite.”
c. “The incubation period is about 1 month.”
d. “The incubation period is about 1 year.”
 
Situation: Mariel Balio, a 12-year-old girl was told by her mother to clean their backyard & burn the
leaves of the jackfruit tree. After 5 days, Mariel experienced fever, body malaise &  occasional petechial
hemorrhages. She was brought to the Rural health unit.
29. Upon seeing Mariel, which of the following questions would the nurse ask to obtain history of the
illness?
            a. “Why didn’t you show to your mother the skin lesions right away?”
            b. “Don’t you have a maid to clean your backyard?”
            c. “Do you have a communicable disease?”
            d. “Were you bitten by mosquitoes when cleaning your backyard?”
30. What is the vector of the patient’s illness?
            a. Female anopheles mosquito
            b. housefly
            c. cockroaches
            d. Aedes aegypti
31. Which of the following is immediately performed to Mariel which serves as the screening test for
dengue hemorrhagic fever?
            a. ELISA
            b. Rumpel-leede test
            c. Complete blood count
            d. Sedimentation rate
32. The nurse’s primary concern during her care to Mariel is one of the following.
            a. report to authority
            b. monitor vital signs
            c. conduct research
            d. case finding
33. The community physician ordered CBC to Mariel. As the nurse taking care of the patient, your
expected CBC finding in patients with dengue hemorrhagic fever is
            a. Increased WBC & decreased RBC
            b. Decreased hematocrit & increased platelet    
            c. Decreased platelet & increased hematocrit
            d. Increased hematocrit & decreased hemoglobin
34. The best measure to control spread of Mariel’s illness is to
            a. avoid hanging clothes in the backyard
            b. keep water container open
            c. spay insecticides once a month
            d. eliminate breeding places of vector
35. Mr. Michael Bautista had a recent travel to Palawan & is suspected to have malaria. The most
important diagnostic test in malaria is
            a. Erythrocyte sedimentation rate
            b. WBC count
            c. Peripheral blood smear
            d. Culture
36. The nurse is reviewing the chart of Mr. Bautista. Which of the following objective cue can be seen in
the patient’s chart?
            a. splenomegaly
            b. Leucocytosis
            c. Increased platelet count
            d. Erythrocytosis
37. Nursing care of patients with Malaria include provision of comfort, avoidance of chills, proper
elimination & accurate medication. Which of the following is most important in preventing spread of
infection?
            a. eradicating breeding sites of vector
            b. educate patient regarding compliance with drug regimen
            c. isolation of patient
            d. avoid sharing utensils
38. A community health nurse teaches a mother comfort measures for her six-year-old child who has
varicella-zoster virus. Which of the following actions, if taken by the mother, requires further
intervention?
            a. Applying cortisone-based cream to the child’s lesion
            b. Patting the child’s lesions with calamine lotion
            c. Bathing the child with tepid sponge bath
            d. Trimming the child’s fingernails very short
39. When teaching a mother about communicable diseases, the nurse informs her that chickenpox is
            a. communicable until all the vesicles have dried
            b. no longer communicable after a high fever has subsided
            c. not communicable as long as the vesicles are intact and surrounded by a red areola
            d. not communicable at all
 
40. The nurse manager is planning the clinical assignments for the day.  The nurse manager avoids
assigning which of the following staff members to the client with herpes zoster?
a.       the nurse who never had mumps
b.       an experienced nurse who never had chickenpox
c.       the nurse who never had roseola
d.       the nurse who never had german measles
41. A nurse is performing an assessment on a 3 year old child with chicken pox.  The child’d
mother tells the nurse that the child keeps scratching at night, and the nurse teaches the mother
about measures that will prevent an alteration in skin integrity.  Which statement by the mother
indicates that teaching was effective?
A. “ I will apply generous amounts of a cortisone cream to prevent itching.”
B. “ I need to place white gloves on my child’s hands at night.”
C. “I need to keep my child ia a warm room at night so that covers will not cause my
child to scratch.”
D. “ I will give my child a glass of warm milk at bedtime to help my child sleep.”
42. Nursing care of patients with measles is avoidance of complications. Which of the following
interventions is done when the child is sensitive to light?
            a. advise to wear dark glasses
            b. cover eyes with a piece of cloth
            c. place patient in a dim, quiet room
            d. shut down windows & door
 
43. A nurse is assessing a 12-month-old infant who recently had fever, runny nose, cough, and white
spots in the mouth for 3 days. A rash developed that started on the face and spread to the whole body. The
nurse should suspect that the child has
            a. rubella
            b. rubeola
            c. varicella
            d. scarlet fever
44. The nurse is aware that rubeola often causes children to have
            a. a maculopapular rash
            b. a paroxysmal cough
            c. an enlarges parotid gland
            d. a generalized vesicular lesion
 
45. A child with rubeola (measles) is being admitted to the hospital. In preparing for the admission of the
child, a nurse plans to place the child on which precautions?
                        a. contact                                              c. respiratory
                        b. enteric                                              d. protective
46. A nurse is caring for a child diagnosed with rubeola.  The nurse notes that the physician has
documented the presence of Koplik spots.  Based on this documentation, which of the following
would the nurse expect to note on assessment of the child.
A. petechiae spots that are reddish and pinpoint on the soft palate.
B. Whitish vesicles located across the chest
C. Small, blue-white spots with a red base found on the buccal mucosa
D. Pinpoint petechiae noted on both legs
Situation:  James, a 4-year-old boy is brought to the Rural Health Unit for cough, fever and rashes.  The
doctor diagnosed him with measles infection.
47. Which of the following should you do first?
a. Perform nursing procedures                   c. Assess the patient
b. Plan for your care regimen                                 d. Identify nursing diagnosis
48. Which of the following procedure should you teach James’ sister when James has a fever at home?
a. proper positioning                                  c. Hot sitz bath
b. Isolation technique                                 d. Tepid sponge bath
49. Which of the following is the most common complication?
a. Meningitis                                             c. Otitis media
b. Pneumonia                                            d. Laryngitis
50. Which nursing consideration must be rendered to avoid complication of pneumonia?
a. Protect eyes from glare of sunlight         c. Ventilated room but free from drafts
b. Correct technique of sponge bath           d. Check prescribed medications faithfully
51. Nutrition is sometimes neglected due to patient’s inability to ask for food.  What is your “meal plan”
for kids with measles?
a. ice cream and cookies                           c. KFC Chicken
b. Fluids and crackers                                d. Mcdonald’s sundae
Situation: Madam Auring, a 25-year-old female sought consultation to a health center after having missed
one menstrual cycle period. It is confirmed that she is 6 weeks pregnant.
52. Further examination showed that Madam Auring is susceptible to rubella. When would it be most
appropriate for her to receive rubella immunization?
a. Immediately                                             c. within 2 weeks before EDC
b. during 2nd trimester                                   d. During her postpartum hospitalization
53. Rubella, if acquired by a pregnant patient during the pregnancy may result in congenital defects in
newborn which includes:
1. deaf-mutism                    2. congenital cataract                
                    3.microcephally                 4. congenital heart defect
 
a.       1,2 & 4
b.       2,3& 4
c.       1, 3, & 4
d.       1, 2, 3 & 4     
e.        
54. The following diseases are characterized by the appearance of skin rashes caused by a virus except:
a. measles                                                   c. variola
b. chickenpox                                               d. scarlet fever
 
55. A child with rubella should be isolated from a/an
a. 20-year-old brother living in the same house
b. 3-year-old girl who lives next door
c. 12-year-old sister who had rubeola during childhood
d. 18-year-old female cousin who recently got married
 
 
Situation:  The patient has a sore that does not heal.  Upon examination it reveals leprosy.
56. Mode of transmission of leprosy is
a. Sharing food                                       c. Borrowing utensils
b. Sexual contact                                    d. Prolonged skin to skin contact
57. Late sign of leprosy in male patients is enlargement of the breast known as:
a. Clawing                                            c. Madarosis
b. Lagopthalmos                                    d. Gynecomastia
58. Your role in prevention of leprosy is
                        a. Health Education                               c. Personal Hygiene
b. Proper nutrition                                  d. BCG vaccination
59.  Multi-drug therapy is being implemented by DOH. Where is the initial treatment done?
                        a. Midwife residence                             c. baranggay captain office
b. barangay health center                                   d. rural health unit
 
60. Which of the following is/are mild leprae reaction?
1. Edema                     
2. Sudden increase in the number of lesions
3. Tenderness on the nerve
4. Sudden paralysis
a. 1, 2 & 3 are correct                          
b. 1 & 3 are correct      
c. 2 & 4 are correct
d. 4 only
e. all of the above
 
61.  Nursing care of patients with Diphtheria is provision of complete bed rest. Which nursing
intervention must be done to a child with laryngeal diphtheria?
            a. ice chips every hour
            b. cookies & candies for snacks
            c. Crackers for lunch
            d. small & frequent feeding
 
62. The characteristic lesion of diphtheria is a grayish-white membrane which can block breathing &
swallowing. Which of the following pertains to this membrane?
            a. Koplik’s spots
            b. Forscheimer spots
            c. Papule
            d. Pseudomembrane
63. The bacteria which causes diphtheria is also known as
            a. Spirochete
            b. Klebs-Loeffler bacillus
            c. Cholera
            d. Amoeba
 
Situation: Camille, a 4-year-old girl was brought to a hospital due to paroxysms of cough. Initial
assessment of the physician was pertussis 
64. Which of the following accurately describes symptoms of pertussis?
1. successive coughing
2. vomiting
3. whooping followed by paroxysms of cough
4. barking cough.
      a. 1, 2 and 3                 c. 2 & 4                        e. all of the above
      b. 1 & 3                                    d. 4 only
65. Which laboratory procedure is done to diagnose pertussis?
              a. Saliva examination                                            c. gram staining of nasopharyngeal
secretion
              b. Culture of nasopharyngeal swab             d. blood culture
66. When blood is examined which blood component is elevated in pertussis?
              a. Eosinophils                                            c. RBC
              b. Lymphocytes                                         d. neutrophils
67. A clinic nurse provides instructions to a mother regarding the care of her child who is
diagnosed with croup.  Which statement by the mother indicates a need for further instructions?
  a“ I will place a cool mist humidifier next to my child’s bed.”
b.“Sips of warm fluids during a croup attack will help.”
c.       “ I will give Tynelol for the fever.”
d.      “ I will give cough syrup every night at bedtime.”
Situation: Mr. Ferdinand Lu, a 35-year-old executive is confined in the Medical ward due to pneumonia.
68. After performing your physical assessment  to the patient, which of the following signifies that he is
having pneumonia?
            a. chest auscultation revealing bronchio-vesicular breath sounds
            b. equal chest expansion occurring every inspiration
            c. tactile fremitus is decreased over the affected area
            d. percussion is resonant on the intercostals spaces
69. Steam inhalation was ordered by the physician. The action of this is to
            a. dilate the bronchioles, thus increasing the viscosity of secretions
            b. reduce the viscosity of secretions
            c. increase the viscosity of secretions
            d. inhibit the growth of microorganism
70. Sputum examination for culture & sensitivity was ordered. This would ascertain which of the
following facts?
            a. the correct analgesic to minimize his chest pain during inspiration
            b. the right antibiotic which would be most curative
            c. the right drug to minimize secretions
            d. the correct position during coughing
71. The following are the nursing guidelines for chest physiotherapy. Which of the following should not
be applied to Mr. Lu?
            a. perform postural drainage at least 1 hour after meals
            b. teach deep breathing & coughing exercises
            c. allow patient to continue postural drainage despite feelings of discomfort
            d. avoid chest tapping in areas of severe pain
 
72. a client with pneumonia has a temperature of 102.6 deg F, is diaphoretic, and ahs a
productive cough.  The nurse include which of the following measures in the plan of
care?
a. Position changes every 4 hours
b.      sunctioning to clear secretions
c.       Frequent linen changes
d.      Frequent offering of bed pan
 
73. Bed rest is prescribed for a client with pneumonia during acute phase of the illness. 
Bed rest serves which of the following purposes?
a.       it reduces the cellular demand for oxygen
b.      it decreases the episodes of coughing
c.       it promotes safety
d.      it promotes clearance of secretions
 
74. The cyanosis that accompanies bacterial pneumonia is primarily caused by which of
the following?
a. decreased cardiac output
b.pleural effusion
         c.inadequate peripheral circulation
d.decreased oxygenation of the blood
 
75. A client with pneumonia is experiencing pleuritic chest pain.  Which of the following
describes pleuritic chest pain?
a. a mild but constant aching in the chest
b. severe midsternal pain
c .moderate pain that worsen on inspiration
d .muscle spasm pain that accompanies coughing
 
76.Which of the following measures would most likely be successful in reducing pleuritic
cheast pain in client with pneumonia?
a. encourage the client to breath slowly
b.have the client practice abdominal breathing
c.offer the client incentive spirometry
d.Teach the client to splint the ribcage when coughing
 
77. Which of the following mental status changes may occur when a client with
pneumonia is first experiencing hypoxia?
a. coma
b. apathy
c. Irritability
d. Depression
78. A client with pneumonia has a temperature ranging between 101 and 102 deg F and periods
of diaphoresis.  Based on this information, which of the following nursing interventions would
be priority?
            A.  Maintain Complete bed rest
            b.  administer oxygen therapy
            c.  provide frequent linen changes
            d.  provide fluid intake of 3 liters/day
 
79. Which of the following would be appropriate expected outcome for an elderly client
recovering from bacterial pneumonia?
a.       A respiratory rate of 25 to 30 breaths/ min
b.      The ability to perform activities of daily living
c.       A maximum lost of 5 to 10 pounds of body weight
d.      Chest pain that is minimized by splinting the ribcage.
 
80. A community health nurse is conducting an educational session with community members regarding
tuberculosis. The nurse tells the group that one of the first symptoms associated with tuberculosis is
a.       a bloody, productive cough
b.       a cough with the expectoration of mucoid sputum
c.       chest pain
d.       dyspnea
81. A nurse performs an admission assessment on a client with a diagnosis of tuberculosis. The nurse
reviews the results of which diagnostic test that will confirm this diagnosis?
                        a. bronchoscopy                                                c. sputum culture
                        b. chest x-ray film                                 d. tuberculin skin test
82.  A client who is HIV (+) has had Mantoux skin test. The nurse notes a 7-mm area of induration at the
site of the skin test. The nurse interprets the result as what?
                        a. positive                                             c. inconclusive
                        b. negative                                            d. it needs repeat testing
83.  A nurse in the out-patient department performed Mantoux skin test today (Tuesday) to an adult client.
What statement made by the client indicates that he understood the instruction well?
a.       “I will come back on Saturday for you to read the result.”
b.       “I will come back on Thursday, same time, for you to read the result.”
c.       “I don’t need to come back anymore.”
d.       “I will come back anytime.”
84.  A client exposed to tuberculosis is taking isoniazid (INH) and develops signs & symptoms of the
disease. The client is instructed to add rifampicin & pyrazinamide to the medication regimen. A nurse
explains to the client that the purpose of adding this second medication is
a.       that rifampicin offsets the side effects of isoniazid
b.       to be certain that resistant organisms are eliminated
c.       that these medications potentiate each other
d.       that isoniazid offsets the side effects of rifampicin
85. A nurse is caring for a client diagnosed with tuberculosis. Which assessment, if made by the nurse,
would not be consistent with the usual clinical presentation of tuberculosis and may indicate the
development of a concurrent problem?
                        a. productive cough                               c. chills & night sweats
                        b. anorexia & weight loss                                   d. high-grade fever
86. A nurse has conducted discharge teaching with a client who was diagnosed with tuberculosis. The
client has been taking medication for a week and a half. The nurse evaluates that the client has understood
the information if the client makes which of the following statements?
a.       “I need to continue drug therapy for 2 months.”
b.       “I should not be contagious after at least 2 weeks of medication therapy.”
c.       “I can’t shop at the mall for the next 6 months.”
d.       “I can return to work if a sputum culture comes back negative.”
87. A client has been taking isoniazid (INH) for a month and a half. The client complains to a nurse about
numbness, paresthesias, and tingling in the extremities. The nurse interprets that the client is experiencing
                        a. small blood vessel spasm                    c. impaired peripheral circulation
                        b. hypercalcemia                                               d. peripheral neuritis
88. A client will start a 6-month course therapy with isoniazid (INH). A nurse plans to teach the client to
a.       use alcohol in small amounts only
b.       report yellow eyes or skin immediately
c.       increase intake of cheese & milk
d.       avoid vitamin supplements during therapy
89. A client has been started on long-term therapy with rifampicin. A nurse teaches the client that the
medication
a.       should be double dosed if one dose is forgotten
b.       may be discontinued independently if symptoms are gone in 3 months
c.       should always be taken with food or antacids
d.       causes orange discoloration of sweat, tears, urine, & feces
90. A nurse has given a client taking ethambutol information about the medication. The nurse determines
that the client understands the instructions if the client states to immediately report
a.       gastrointestinal side effects
b.       impaired sense of hearing
c.       orange-red discoloration of body secretions
d.       difficulty in discriminating the color red from green
Situation: Tuberculosis, a primary respiratory disease is common among malnourished individuals living
in crowded places. The DOH survey last 1997 revealed that there are approximately 16 million afflicted
with the disease.
91. The bacteria causing the disease is
            a. Mycobacterium bovis
            b. Mycobacterium tuberculosis
            c. Mycobacterium africanum
            d. Mycobacterium leprae
Situation:  Mang Romy is malnourished living in a squatter’s area.  He came in the health center due to
loss of weight easy, fatigability, chronic cough and low grade fever.
92. The main source of infection pulmonary tuberculosis is:
a. crowded living quarters
b. unavailability of potable drinking water
c.  direct contact with infected person
d. unsanitary surroundings
93. A physician orders an X-ray examination for patient.  What is the purpose for this?
a. To check if patient have relatives with TB
b. To see the evidence of having the disease
c. Determine severity of the lesions in the lungs
d. To aid in doctors prescriptions.
94. The nurse notes 12 mm of induration at the site of a Mantoux test when a client returns to the health
office to have it read. The nurse should explain to the client that this is
            a. Test result is negative, & no follow-up is needed.
            b. Test was used for screening and a TINE test will now be given.
            c. Skin test is inconclusive and will have to be repeated in six weeks.
            d. The result is positive and a need for further tests, including chest x-ray is indicated
95. Staff nurses learn that a patient they have been caring for during the last few weeks has just been
diagnosed with tuberculosis. When the nurses express concern about contracting tuberculosis themselves,
the charge nurse’s response should be based on which of the following statements?
            a. Tuberculosis is easily treated with short course of antibiotics
            b. The Mantoux test is used to confirm diagnosis of tuberculosis
            c. Tuberculosis is not highly infectious when standard precautions are followed
            d. Vaccination with BCG will be used to immunize the nurses against infection
96. Before returning a child who is being treated for tuberculosis to his home, the community health nurse
determines
            a. that the child has a private room
            b. that all family members have been tested
            c. proper room ventilation
            d. home school placement
97. Which of the following statements, if made by a patient who has tuberculosis, would indicate a
correct understanding of disease transmission?
            a. “The disease can be acquired from breast milk.”
            b. “The disease does not spread beyond the lungs.”
            c. “I contracted the disease through bird droppings.”
            d. “I can be infected more than once with this disease.”
98. Which of the following instructions should a nurse give to a patient who has tuberculosis and is
receiving rifampicin?
            a. “Stay out of the sun while you are taking the medication.”
            b. “Expect your urine to turn reddish-orange.”
            c. “Take your medication on an empty stomach.”
            d. “Take your medication until your skin test is negative.”
99. A patient who is taking INH, Rifampicin, PZA and pyridoxine for the treatment of tuberculosis asks a
nurse why the Vitamin B6 (pyridoxine) is necessary. The nurse would respond that
            a. “Vitamin B6 is necessary for the absorption of INH.”
            b. “Vitamin B6 activates the metabolism of INH.”
            c. “INH leads to Vitamin B6 depletion, which leads to neurotoxic effects.”
            d. “INH can cause anorexia, which leads to vitamin B6 depletion.”
100. What is the rationale that supports multidrug treatment for clients with tuberculosis?
a.       Multiple drugs potentiate the drug actions
b.      Multiple drugs reduce undesireable side effects
c.       Multiple drugs allow reduced dosages to be given
d.      Multiple drugs reduce development of resistant strains of the bacteria
 
101. The client with TB is to be discharged home with community health nurse follow-up.  Of
the following interventions, which would have the highest priority?
a.       offering the client emotional support
b.      teaching the client about the disease and its treatment
c.       coordinating various agency services
d.      Assessing the client’s environment for sanitation
 
102. The nurse should caution sexually active female clients taking INH that the drug has which
of the following effects?
a.       Increases the risk of vaginal infection
   b. Has mutagenic effects on ova
c. Decreases the effectiveness of oral contraceptives
d. Inhibits ovulation
 
103.Clients who have had active TB are at risk for recurrence.  Which of the following
conditions increases that risk?
b.      Cool and damp weather
c.       Active exercise and exertion
d.      Physical and emotional stress
e.       Rest and inactivity
 
104.The nurse should include which of the following instructions when developing a teaching
plan for clients who are receiving INH and rifampicin for treatment of TB?
a. Take the medications with antacids
b. Double the dosage if a drug dose is forgotten
c. Increase intake of dairy products
d. Limit alcohol intake
 
Situation:  Mandy started to complain severe headache and high grade fever for almost two weeks.  She
remembered eating “sapin sapin” in a nearby store.  The initial impression of the admitting doctor was
typhoid fever.
105. The organ of predilection of typhoid infection is:
a. small intestine                                           c. gallbladder
b. colon                                                       d. Spleen
106.  Mandy reported presence of black or tarry stool a day prior to admission.  Based on the natural
history of disease, she is on what stage of the disease process?
a. prodomal satge                                         c. defervescence stage
b. fastidial stage                                           d. recovery stage
107. Nurse Arianne examined Mandy.  She was trying to look for the an exanthema of typhoid fever
which is
a. maculopapular rashes on trunk                  c. bluish discoloration on arms
b. petechial rashes on extremities                  d. migratory painless nodules
108. The confirmatory test for typhoid on the first  week of illness is
a. Widal test                                                            c. Typhidot
b. Urine culture                                            d. Blood culture
109. The stool culture of a child with profuse diarrhea reveals Salmonella bacilli.  After teaching
the mother about the course of Salmonella enteritis, which of the following statements by the
mother indicates effective teaching?
a.“Some people become carriers and stay infectious for a long time”
   b.“ After the acute stage passes the organism is usually not present in the stool.”
   c.“ Although the organism may be alive indefinitely, in time it will be of
no danger to anyone.”
   d.“ If my child continues to have the organism in the stool, an antitoxin can help
        destroy the organism.”
 
110. Which of the following would be the best activity for the nurse to include in the plan of care
for an infant experiencing severe diarrhea?
a.       Monitoring the total 8 hour formula intake
b.      Weighing the infant each day
c.       Checking the anterior fontanel every shift
d.      Monitoring abdominal skin turgor every shift
 
111. Which of the following would be most appropriate for the nurse to teach the mother of a 6
month old infant hospitalized with severe diarrhea to help comfort her infant who is fussy?
 
a. Offering a pacifier
b.Placing a mobile above the crib
c. Sitting at the crib side talking to the infant
d.Turning the television on to cartoons
 
112.Which of the following nursing diagnosis would be appropriate for the nurse to identify as a
priority diagnosis for an infant just admitted to the hospital with a diagnosis of gastroenteritis?
a.Pain related to repeated episodes of vomiting
b.Deficient Fluid volume related to excessive losses from severe diarrhea
c.Impaired parenting related to infant’s loss of fluid
d.Impaired Urinary Elimination related to increased fluid intake feeding
pattern
 
113.Which of the following would the nurse use to determine achievement of the expected
outcome for an infant with severe diarrhea and nursing diagnosis of Deficient Fluid volume
related to passage of profuse amounts of watery diarrhea?
a. Moist mucous membranes
b.Passage of a soft, formed stool
            c.Absence of diarrhea for a 4 hour period
d.Ability to tolerate intravenous fluids well
 
114.A child is admitted to the pediatric unit with the diagnosis of severe gastroenteritis.  Which
of the following would be most appropriate for the nurse to do?
a. Institute standard precautions
b. Place the child in a semiprivate room
c. Use regular eating utensils
d. Single- bag all linens
 
115. Of the following would most likely alert the nurse to the possibility that a preschooler is
experiencing moderate dehydration?
a. Vomiting
b. Diaphoresis
c. Absence of tear formation
d. Decreased urine specific gravity
 
116.The physician orders IV fluid replacement therapy with potassium chloride to be added for a
child with severe gastroenteritis.  Before adding the potassium chloride to the IV, which of the
following assessments would be most important?
a. Ability to void
b. Passage of stool today
c. Baseline echocardiogram
d. Serum calcium level
 
117.Which of the following would first alert the nurse to suspect that a child with severe
gastroenteritis who has been receiving IV therapy for the past several hours may be developing
circulatory overload?
a. a drop in blood pressure
b. change to slow, deep respirations
c. auscultation of moist crackles
d. marked increase in urine output
118. To help prevent an infant from contracting infectious diarrhea, the nurse should instruct the mother
on which topic?
            a. When to introduce solid foods into the infant’s diet
            b. How to prepare, handle & store infant formula
            c. When to have the child immunized
            d. When to bring the infant to the clinic for routine check-ups
119. A nursing student is assigned to admit to the hospital a child who has been experiencing vomiting &
diarrhea. A physician establishes a diagnosis of acute gastroenteritis & isotonic dehydration. A nursing
instructor asks the student to describe isotonic dehydration. The nursing student responds accurately by
telling to the instructor that isotonic dehydration
a.       occurs when water & electrolytes are lost in about the same proportions as they
exist in the body
b.       occurs when the loss of electrolytes is greater than the loss of water
c.       occurs when the loss of water is greater than the loss of electrolytes
d.       causes the serum sodium level to rise above 150 mEq/L
120. A clinic nurse is assessing a child for dehydration. The nurse determines that the child is moderately
dehydrated if which symptom is noted on assessment?
                        a. flat fontanels                                     c. moist mucous membranes
                        b. pale skin color                                               d. oliguria
121. Bloody & mucoid stool associated with tenesmus is pathognomonic of
                        a. diphtheria                                          c. bacillary dysentery
                        b. violent dysentery                                d. amoebic dysentery
 
122. The intermediate vector of schistosomiasis is
a. mosquitoes                                                           c. rats
b. flies                                                                     d. fresh water snail
 
123. Diet of patient with acute parotitis should contain
            a. High Na, High CHO, High fats
            b. Low Na, Low CHO, Low fats
            c. High CHO, High fats
            d. High CHO, Low fats
 
124.A mother is worried because her 5-year-old son acquired mumps infection. She asks about the most
severe complication of this infection. The best response of the nurse should be
a.       “You don’t have to worry because there is no known complication of mumps.”
b.       “Although rare, the most severe complication of mumps is encephalitis.”
c.       “Your son will become sterile for life.”
d.       “Your child may develop pneumonia.”
 
Situation: Mr. Reyes,  25-year-old clerk is admitted to the hospital with a chief complaint of easy
fatigability associated with fever, amber-colored urine & clay colored stools. He is diagnosed with Acute
Viral Hepatitis type B. On the fourth hospital day, he developed jaundice & strongly insisted that his
visitors be restricted to her immediate family members.
125. Potential problems that a nurse may encounter include all of the following except
            a. Bleeding
            b. Pruritus
            c. Weight loss
            d. Hyperglycemia
126 Based on the data given, one of the nursing diagnosis is
            a. Depression related to feelings of guilt
            b. Anxiety related to feeling of impending doom
            c. Body image disturbance related to altered skin appearance
            d. Anger & hostility related to restriction of physical activity
127. The nurse observes that Mr. Reyes has clay-colored stools. He asked you why is this so. The reason
for this is that
            a. Hepatic uptake of bilirubin is impaired
            b. Excretion of fecal urobilinogen is increased
            c. Conjugated bilirubin reenters the bloodstream
            d. Excretion of conjugated bilirubin into the intestines is decreased
128. To best reduce the risk of spread of Mr. Reyes’ disease, the nurse must observe the proper disposal
of the patient’s
            a. Urine & feces
            b. Nasogastric secretions
            c. Used needles & syringes
            d. Feces & oral secretions
 
129.The nurse plans care for the client with Hepatitis A with the understanding that the causative
virus will be excreted from the client’s body primarily through the:
a. skin
b. feces
c. urine
d. blood
 
130. The nurse is planning a staff development program for health care staff on how to care for
the clients with hepatitis A.  Which of the following precautions would the nurse indicate as
essential when caring for clients with Hepatitis A?
a. Gowning when entering aclient’s room
b. Wearing a mask when providing care
c. Assigning the client to a private room
d. Wearing gloves when giving direct care
 
131. When developing a plan of care for the client with viral hepatitis, the nurse should
incorporate nursing orders that reflect the primary treatment.  Emphasis will be on ensuring that
the client receives which of the following?
a. adequate bed rest
b. generous fluid intake
c. regular antibiotic therapy
d. daily IV theraphy
 
132.In a client with viral hepatitis, the nurse would closely assesses for indicators of which of the
following abnormal laboratory values?
a. prolonged PT
b.decreased blood glucose level
c.elevated serum potassium
d.decreased serum calcium
 
133. Which of the follwing diets would most likely be prescribed for a client with viral hepatitis?
a. high fat, low protein
b. high protein, low carbohydrates
c.high crbohydrates, high calorie
d. low sodium, low fat
134. The nurse develops a teaching plan for the client about how to prevent the transmission of
hepatitis A.  Which of the following discharge instructions is appropriate for the client?
a. Spray the house to eliminate infected insects
b. Tell the family members to try to stay away from the client.
c. Tell the family members to wash their hands frequently
d.  Disinfect all clothing and eating utensils
 
135. The nurse assesses that the client with hepatitis is experiencing fatigue, weakness, and a
general feeling of malaise.  The client tires rapidly during morning care .  Based  on this
information, which of the following would be an appropriate nursing diagnosis?
a. Impaired physical mobility related to malaise
b. Self-cared Deficit related to fatigue
c. Ineffective Coping related to long term illness
d. Activity Intolerance related to fatigue
 
136. A client has been admitted to the hospital with a diagnosis of hepatitis B.  the client tells the
nurse, “ Ifeel so isolated from my friends and family.  Nobody wants to be around me.”  What
would be the most appropriate nursing diagnosis for this client?
a. Anxiety related to feelings of isolation
b. Social isolation related to significant other’s fear of contracting disease
c. Powerlessness related to lack of social support
d. Low-self Esteem related to feelings of rejection.
137. A delivery room nurse was unable to perform newborn care to a neonate. In the history of the
postpartum mother, she recently was diagnosed with gonococcal infection that was untreated. Which of
the following will most likely to occur in the neonate?
a. The neonate will have bleeding problems due to lack of vitamin K
b. The neonate will develop ophthalmia neonatorum
c. The neonate will not have any medical problems
d. The neonate will develop respiratory failure
138. Cristina Yap, a 30-year-old female came to a clinic due to burning sensation in the genital area. The
physician diagnosed her with non-gonococcal urethritis. The nurse knows that the most likely etiologic
agent of non-gonococcal urethritis is
a. Neiserria gonorrhea                               c. Chlamydia trachomatis
b.Trichomonas vaginalis                            d. Haemophilus ducreyi
 
Situation:  A 24-year-old male presents to the OPD with a 3 day history of burning sensation on urination
which progressed to the development of purulent, yellowish-to-greenish urethral discharge.  The nurse
suspects of STD.
139. On laboratory examination, the diagnosis of gonorrhea is confirmed by which of the following on
the report of the staining result?
a. Gram negative intracellular diplocci                        c. Numerous spirochetes
b. Gram positive extracellular diplococci        d. Gram negative extracellular diplococci
 
 
140. The nurse knows that the easiest factor to break in the infectious process to prevent STD spread
would be
a. Reservior                                                 c. Agent
b. Mode of transmission                                d. Portal of entry
141. The prevention and control of STD is made difficult by
a. Lack of drugs specific for STD                 c. No government program on STD
b. Self-medication of infected persons                       d. Lack of  better laboratory facility
142. Nurse is very active in contact tracing and epidemiologic treatment of all gonococcal identified
contacts to prevent
a. Development of resistant strain                 c. Non venereal transmission
b. Asymptomatic infection                            d. Re-infection
143. The patient made the following statement to the nurse.  Which statement would need reinforcement
of health teachings?
a. I need to take drugs for prophylaxis the next time I do it
b. Gonorrhea does not confer lifelong immunity
c. The disease can also be transmitted through other means
d. I will finish the prescribed medication to prevent resistance of microorganism
 
144. A female client with gonorrhea informs the nurse that she had sexual intercourse with her boyfriend
and asks the nurse “Would he experience any symptoms? The nurse responds that in males, the initial
manifestation is
            a. Impotence
            b. Scrotal swelling
            c. Sterility
            d. Dysuria
Situation: Mr. Larry Hipolito, a 25 y/o male came to an out-patient clinic complaining of pain over the
hypogastrium & yellowish discharge from his urethra. A diagnosis of gonorrhea is made.
145. Since Larry has gonorrhea, he most likely has which of these symptoms?
            a. a maculo-papular rash on the pubic area
            b. a painless ulcer on the penis
            c. wart-like lesions on the scrotum
            d. pain upon urination
146. Larry was instructed to remain in the clinic for at least 30 minutes so that adverse reactions to the
medication can be detected. Which of these comments made by Larry is indicative of an adverse reaction
to the specific drug?
            a. “My chest feels tight”
            b. “I need to go to the bathroom.”
            c. “I need a glass of water.”
            d. “My hands are shaky.”
147. Larry asked for possible complications if he had not received treatment for gonorrhea. He would be
told that one such complication is
            a. liver atrophy
            b. epididymis
c. peripheral vascular disease
d.  ectopic pregnancy
148. Characteristic lesion of primary syphilis is
            a. chancroid
            b. condyloma lata
            c. gumma
            d. chancre
149. A client is suspected of having late-stage syphilis. When obtaining a health history, the nurse
recognizes that the statement by the client that would most support this diagnosis would be
            a. “I noticed a wart on my penis.”
            b. “I have sores all over my mouth.”
            c. “I have been losing a lot of hair lately.”
            d. “I’m having trouble keeping my balance.”
Situation: Mark Smith, a 37 y/o male has 19 years history of IV drug abuse. He has poor appetite, nausea,
& weight loss of 10 kg in the past 6 months. A diagnosis of HIV was made.
150. To initiate a nursing care plan for Mr. Smith, the nurse should consider that a depressed immune
system is caused by which of these pathophysiological alterations?
            a. Decreased leucocyte count
            b. low platelet count
            c. reduction of  nucleated RBC
            d. Increased sedimentation rate
151. Lesions that will characterize Mr. Smith is in the advanced stage of the illness is
            a. condyloma lata
            b. gumma
            c. Kaposis sarcoma
            d. Pneumocyctis carinii pneumonia
 
152. A patient who is HIV positive is admitted to an isolation unit for treatment of tuberculosis. The nurse
observes that the patient has difficulty following instructions, short-term memory loss & difficulty
concentrating. The nurse should be aware that these behaviors
            a. will disappear when isolation is no longer required
            b. are symptoms of AIDS dementia complex
            c. are normal for the early stage of infection
            d. are a result of mycobacterium infection
153. A patient who has AIDS develops Pneumocyctis carinii pneumonia (PCP). The patient asks the
nurse, “How did I get this pneumonia?” The nurse’s response should be based on which of these
statements about PCP?
            a. It occurs in immunosuppresed persons from proliferation of organisms that are
            normally present  in the body
            b. It is transmitted from close contact with an infected individual who has a suppressed
            immune system.
            c. It results from exposure to a carrier of the organism who has not taken appropriate
            precautions.
            d. It is most often acquired from unprotected sex with an infected individual.
154. An HIV-positive patient is prescribed zidovudine and ganciclovir. The nurse should be aware that
both drugs have which of the following side effects?
            a. Peripheral neuropathy
            b. Insomnia
            c. Anticholinergic effects
            d. Bone marrow suppression
155. Blood screening tests of the immune system of a client with AIDS would indicate
            a. a decrease in CD4 T cells
            b. an increase in thymosine (thymic hormone)
            c. an increase in immunoglobulin E
            d. a decrease in erythrocytes
156. Mr. Santos, a 48-year-old male is an HIV suspect admitted in San Lazaro. In assessing the patient’s
condition for possible HIV infection, the least expected symptom to occur is
            a. Night sweating
            b. Fever
            c. Fungal infection in the mouth
            d. Acute diarrhea
157. The nurse is caring for the client with acquired immunodeficiency syndrome. The nurse detects early
infection with Pneumocystis carinii by monitoring the client for which clinical manifestation?
a.       dyspnea on exertion                         c. fever
b.       dyspnea at rest                                d. cough
158. The nurse is caring for the client with AIDS who is experiencing night fever & sweats. Which
nursing intervention would be the least helpful in managing this symptom?
a.       keep a change of bed linens nearby in case they are needed
b.       administer an antipyretic after the client has spike of fever
c.       make sure the pillow has a plastic cover
d.       keep liquids at bedside
159. The client exposed to human immunodeficiency virus about 3 months ago has seroconverted to an
HIV-positive status. The nurse anticipates that the client will experience which of the following at this
time?
                        a. oral lesions                                        c. purplish skin lesions
                        b. chronic cough                                                d. no signs & symptoms
160. The client with AIDS has raised dark purplish-colored lesions on the trunk of the body. The nurse
anticipates that which of the following procedures will be done to confirm whether these lesions are due
to Kaposi’s sarcoma?
                        a. Enzyme-linked immunosorbent assay  c. western blot
                        b. skin biopsy                                        d. lung biopsy
 
 
 
CD- COMPRE EXAM 1
 KEY ANSWER
 
 
1. A nursing instructor asks a nursing student to describe live or attenuated vaccines.  The
student tells the instructor that these types of vaccines are:
a. vaccines that have their virulence (potency) diminished so as to not produce a full-
blown clinical illness.
b.  vaccines that contain pathogens made inactive by either chemical or heat.
c.  bacterial toxins that have been made inactive by either chemicals or heat.
d.  vaccines that have been obtained from the pooled blood of many people and provide
antibodies to a variety of diseases.
 
    2.  Passive immunization is given as a preventive measure when
a. Routine immunization is given
b. Clinical recovery is on the way
c. Exposure to a frank case has been observed
d. Symptoms of the disease is manifested by the patient
 
   3.  A 6-month-old infant receives a DPT immunization at a well-baby clinic. The mother returns
        home and calls the clinic to report that the infant has developed swelling & redness at the    
        site of injection. The nurse tells the mother to
a.       apply a warm compress to the injection site
b.       bring the infant back to the clinic
c.       apply an ice pack to the injection site
d.       monitor the infant for a fever
   4.  A nursing student is assigned to administer immunizations to children in a clinic. The
        nursing instructor asks the student about the contraindications to receiving an immunization.
        The student responds correctly by telling the instructor that a contraindication to receiving   
         an immunization is if a child has
                        a. a cold                                                            c. otitis media
                        b. mild diarrhea                                                 d. a severe febrile illness
  5. A 12-month-old received immunizations at his well-child visit. Fourteen days later, the
      mother reports that the child has red maculopapular rash on the trunk & extremities. This rash
      is most likely due to which of the following vaccines?
            a. DPT
            b. OPV
            c. MMR
            d. Hib
6. When a woman receives rubella vaccination, a nurse would give her which of the following
instructions?
            a. “Don’t get pregnant for at least 3 months.”
            b. “Refrain from eating eggs & egg products for 48 hours.”
            c. “Limit contact with visitors for at least one week.”
            d. “Avoid breast-feeding the baby for at least 24 hours.”
7. A 12 years old child comes into the clinic for the MMR vaccines for entry into the 6 th grade the nurse
asks the mother if the child had changes in health since last seen. Which statement by the mother would
indicate a potential contraindication to MMR immunization?
a. “ My child had neomycin sulfate for acne and develop hives and difficulty in breathing”
b. After the last MMR my child developed a low grade and had a rash  for the couple of days 
c. My child’s brother is taking corticosteroids for juvenile rheumatoid arthritis
d. My child just had a tuberculin TB test.
8. The parents of an 18-month-old baby who developed signs of tetanus are concerned about how the
disease will affect their child’s intellectual ability in the future. The nurse’s best response will be
a. “The child’s intellectual functioning may be damaged.”
b. “The child’s intellectual functioning should remain intact.”
c. “The child’s intellectual functioning may be temporarily retarded.”
d.  “The child’s intellectual functioning depends on the severity of complications.”
9. A 7 day old child is brought to the health center because of inability to suck since yesterday. You
suspect that this is a case of neonatal tetanus. As you asses the child which of the questions will you ask
the mother in order to confirm your suspision?
            a. Did the child normally suck and cry 2-4 days before she was brought to the center ?
            b. Did the child have convulsion?
            c. Have you observed stiffness of the child’s body
            d. all of these questions
10. Which of these statements about tetanus is wrong?
            a. immunity last for 5 months in infants born to immune mother
b. Immunity can be obtained after two primary doses of toxoid in mothers one moth before
delivery.
            c. recovery from tetanus does not result in solid immunity
            d. tetanus bacillus is directly transmitted from man to man.        
11. Prevention of tetanus neonatorum can be achieved through:
a. increasing the immunization coverage of women of child-bearing age with tetanus toxoid.
            b. immunizing newborns
            c. increasing the proportions of deliveries attended by the traditional birth attendants
            d. all of the above
12. If you were to conduct on the seminar on the prevention and control of tetanus neonatorium for your
new midwives, which of the following should be given emphasis?
            a. Methods equipments and techniques of asepsis in childbirth
            b. Education of mothers on the practice of asepsis in the care of umbilical stamp.
            c. Education of birth attendants on the practice of strict
            d. All of these measures
Situation:  A 7 year old boy was admitted in the hospital because of convulsion 6 hours  ago.  A week ago
the child experienced on and off cough and low grade fever.  On examination, the nurse observes nuchal
rigidity.
13. The most common cause of bacterial meningitis in children is
a. Haemopilus influenzae B                                    c. Neisseria meningitides
b. Streptococcus pnemoniae                                   d. Meningococcus
14. To confirm the diagnosis, the doctor performed lumbar puncture.  The result that would indicate that
the cause of the disease is more of bacterial in origin would be
a. Increased pressure, glucose, and protein
b. Increased pressure, protein and decreased glucose
c. Increased protein, low glucose, and pressure
d. Cloudy CSF, increased protein and glucose
e.       A mother brings the child to the health care clinic. 
15. The child has been complaining of severe headaches and has been vomiting.  The child
has a high fever, and the nurse notes the presence of nuchal rigidity in the child.  The nurse
suspects a possible diagnosis of bacterial meningitis. The nurse continues to assess the child
for the presence of Kernig’s sign.  Which finding would indicate the presence of this sign.
a. Inaability of the child to extend the legs fully when lying  supine
b. Flexion of the hips when the neck is flexed from a lying position
c. Pain when the chin is pulled down to the chest
d. Calf pain when the foot is dorsiflexed
 
16. A patient who is diagnosed with encephalitis is being treated with mannitol. Which of the following
patient outcomes should indicate to a nurse that treatment with mannitol has been effective for a patient
who has increased intracranial pressure?
            a. Decreased level of consciousness
            b. Increased urinary output
            c. Elevated body temperature
            d. Slowed papillary response
 
17. Maria Victoria, a 26-year-old female equestrian, developed fever, chills & body malaise for 5 days.
On the 7th day of illness, she developed convulsions & was rushed to a tertiary hospital. As an ER nurse in
that hospital, you noticed that the patient’s upper & lower extremities are in a flexed position. You very
well know that this is
a. decorticate rigidity                                                c. nuchal rigidity
b. decerebrate rigidity                                               d. opisthotonus
Situation: Mrs. Ferrer delivered a healthy baby girl weighing 6.8 lb. Due to knowledge deficit, the mother
was not able to bring the neonate for immunization. When the child reached 3 years old, she developed
pain on both knees & discomfort during standing and walking prompting her to prefer to sit or lie down.
The child started having fever on the 3 rd day after the leg pain. When the child was brought to a hospital, a
diagnosis of Poliomyelitis was made.
18. During the early symptoms of poliomyelitis, the client usually suffers from
            a. coryza
            b. headache
            c. seizures
            d. vomiting
Situation : Cristina, an 8-year-old girl was never given any kind of immunization because the family had
never visited a health care unit.  Now she is diagnosed to have acute poliomyelitis.
19. The major route of transmission of polio virus in poor sanitation environment is:
      a. Sewage disposal                                           
b. Water system                                                  
            c. Fecal- oral
            d.  Food stuff
20. When the virus is ingested, where does it multiply?
      a. Gastro- intestinal tract                                               
      b. Integumentary system           
      c. Urinary tract
      d. Reproductive tract
21. As a rural health nurse, your most important function in polio prevention is to:
a. Educate the public on the advantages of immunization
b. Distribute leaflets about the causes of polio
c. Call all mother for  lecture
d. Refer all children to the doctor
22. Your health teaching to Cristina, the polio victim includes:
a. Proper waste disposal                                   
b. Proper eating habits  
c. Regular health check-up
d. Advocacy in health
23. The best nursing intervention in the case of Cristina with paralysis of lower extremity due to
poliomyelitis is:
      a. Letting Cristina attend mother class   
      b. Rendering physical comfort
      c. Rendering physical therapy exercises to limit atrophy
      d. Diet instruction
24. Sight & sound of water as well as drip of the faucet disturb a patient with rabies. Which of the
following should the nurse do first in taking care of a patient with rabies?
            a. Place patient near the window
            b. Cover IV bottle with brown paper bag
            c. Give tepid sponge bath
            d. Place the patient near a utility room
25. If a dog bites a person, it should be
            a. Leashed immediately & observed for 10 days
            b. Killed immediately
            c. Brought to a veterinarian for examination to determine the presence of rabies in the
             blood
            d. punished by not giving food & water
26. A 10-year-old boy arrives with his mother at the emergency department after being bitten by a stray
dog. There is a bleeding soft tissue injury on the inner aspect of the left forearm. The first nursing action
would be
            a. Notify the barangay tanod to capture the dog
            b. Ask the mother if her son is allergic to horse serum
            c. assess the injury, vital signs, and past health history
            d. inoculate the child with human rabies immune globulin
 
27. Emilie Lopamia, a 35-year-old female was bitten by a stray dog on the left hand. She sought an advice
of the nurse in the health center. The best response of the nurse is
a. “You should observe the dog for changes in behavior so you will know if the dog is
possibly rabid or not.”
b. “You should receive immunization for rabies immediately.”
c. “You don’t have to worry because it’s just a minor wound.”
d. “You should receive immunization for rabies & tetanus as soon as possible.”
28. Emilie asked the nurse regarding the incubation period of rabies. Which of the following statements
made by the nurse is appropriate?
a. “The incubation period is about 2 weeks.”
b. “The incubation period varies depending on the site of the bite.”
c. “The incubation period is about 1 month.”
d. “The incubation period is about 1 year.”
 
Situation: Mariel Balio, a 12-year-old girl was told by her mother to clean their backyard & burn the
leaves of the jackfruit tree. After 5 days, Mariel experienced fever, body malaise &  occasional petechial
hemorrhages. She was brought to the Rural health unit.
29. Upon seeing Mariel, which of the following questions would the nurse ask to obtain history of the
illness?
            a. “Why didn’t you show to your mother the skin lesions right away?”
            b. “Don’t you have a maid to clean your backyard?”
            c. “Do you have a communicable disease?”
            d. “Were you bitten by mosquitoes when cleaning your backyard?”
30. What is the vector of the patient’s illness?
            a. Female anopheles mosquito
            b. housefly
            c. cockroaches
            d. Aedes aegypti
31. Which of the following is immediately performed to Mariel which serves as the screening test for
dengue hemorrhagic fever?
            a. ELISA
            b. Rumpel-leede test
            c. Complete blood count
            d. Sedimentation rate
32. The nurse’s primary concern during her care to Mariel is one of the following.
            a. report to authority
            b. monitor vital signs
            c. conduct research
            d. case finding
33. The community physician ordered CBC to Mariel. As the nurse taking care of the patient, your
expected CBC finding in patients with dengue hemorrhagic fever is
            a. Increased WBC & decreased RBC
            b. Decreased hematocrit & increased platelet    
            c. Decreased platelet & increased hematocrit
            d. Increased hematocrit & decreased hemoglobin
34. The best measure to control spread of Mariel’s illness is to
            a. avoid hanging clothes in the backyard
            b. keep water container open
            c. spay insecticides once a month
            d. eliminate breeding places of vector
35. Mr. Michael Bautista had a recent travel to Palawan & is suspected to have malaria. The most
important diagnostic test in malaria is
            a. Erythrocyte sedimentation rate
            b. WBC count
            c. Peripheral blood smear
            d. Culture
36. The nurse is reviewing the chart of Mr. Bautista. Which of the following objective cue can be seen in
the patient’s chart?
            a. splenomegaly
            b. Leucocytosis
            c. Increased platelet count
            d. Erythrocytosis
37. Nursing care of patients with Malaria include provision of comfort, avoidance of chills, proper
elimination & accurate medication. Which of the following is most important in preventing spread of
infection?
            a. eradicating breeding sites of vector
            b. educate patient regarding compliance with drug regimen
            c. isolation of patient
            d. avoid sharing utensils
38. A community health nurse teaches a mother comfort measures for her six-year-old child who has
varicella-zoster virus. Which of the following actions, if taken by the mother, requires further
intervention?
            a. Applying cortisone-based cream to the child’s lesion
            b. Patting the child’s lesions with calamine lotion
            c. Bathing the child with tepid sponge bath
            d. Trimming the child’s fingernails very short
39. When teaching a mother about communicable diseases, the nurse informs her that chickenpox is
            a. communicable until all the vesicles have dried
            b. no longer communicable after a high fever has subsided
            c. not communicable as long as the vesicles are intact and surrounded by a red areola
            d. not communicable at all
 
40. The nurse manager is planning the clinical assignments for the day.  The nurse manager avoids
assigning which of the following staff members to the client with herpes zoster?
a.       the nurse who never had mumps
b.       an experienced nurse who never had chickenpox
c.       the nurse who never had roseola
d.       the nurse who never had german measles
41. A nurse is performing an assessment on a 3 year old child with chicken pox.  The child’d
mother tells the nurse that the child keeps scratching at night, and the nurse teaches the mother
about measures that will prevent an alteration in skin integrity.  Which statement by the mother
indicates that teaching was effective?
A. “ I will apply generous amounts of a cortisone cream to prevent itching.”
B. “ I need to place white gloves on my child’s hands at night.”
C. “I need to keep my child ia a warm room at night so that covers will not cause my
child to scratch.”
D. “ I will give my child a glass of warm milk at bedtime to help my child sleep.”
42. Nursing care of patients with measles is avoidance of complications. Which of the following
interventions is done when the child is sensitive to light?
            a. advise to wear dark glasses
            b. cover eyes with a piece of cloth
            c. place patient in a dim, quiet room
            d. shut down windows & door
 
43. A nurse is assessing a 12-month-old infant who recently had fever, runny nose, cough, and white
spots in the mouth for 3 days. A rash developed that started on the face and spread to the whole body. The
nurse should suspect that the child has
            a. rubella
            b. rubeola
            c. varicella
            d. scarlet fever
44. The nurse is aware that rubeola often causes children to have
            a. a maculopapular rash
            b. a paroxysmal cough
            c. an enlarges parotid gland
            d. a generalized vesicular lesion
 
45. A child with rubeola (measles) is being admitted to the hospital. In preparing for the admission of the
child, a nurse plans to place the child on which precautions?
                        a. contact                                              c. respiratory
                        b. enteric                                              d. protective
46. A nurse is caring for a child diagnosed with rubeola.  The nurse notes that the physician has
documented the presence of Koplik spots.  Based on this documentation, which of the following
would the nurse expect to note on assessment of the child.
A. petechiae spots that are reddish and pinpoint on the soft palate.
B. Whitish vesicles located across the chest
C. Small, blue-white spots with a red base found on the buccal mucosa
D. Pinpoint petechiae noted on both legs
Situation:  James, a 4-year-old boy is brought to the Rural Health Unit for cough, fever and rashes.  The
doctor diagnosed him with measles infection.
47. Which of the following should you do first?
a. Perform nursing procedures                   c. Assess the patient
b. Plan for your care regimen                                 d. Identify nursing diagnosis
48. Which of the following procedure should you teach James’ sister when James has a fever at home?
a. proper positioning                                  c. Hot sitz bath
b. Isolation technique                                 d. Tepid sponge bath
49. Which of the following is the most common complication?
a. Meningitis                                             c. Otitis media
b. Pneumonia                                            d. Laryngitis
50. Which nursing consideration must be rendered to avoid complication of pneumonia?
a. Protect eyes from glare of sunlight         c. Ventilated room but free from drafts
b. Correct technique of sponge bath           d. Check prescribed medications faithfully
51. Nutrition is sometimes neglected due to patient’s inability to ask for food.  What is your “meal plan”
for kids with measles?
a. ice cream and cookies                           c. KFC Chicken
b. Fluids and crackers                                d. Mcdonald’s sundae
Situation: Madam Auring, a 25-year-old female sought consultation to a health center after having missed
one menstrual cycle period. It is confirmed that she is 6 weeks pregnant.
52. Further examination showed that Madam Auring is susceptible to rubella. When would it be most
appropriate for her to receive rubella immunization?
a. Immediately                                             c. within 2 weeks before EDC
b. during 2nd trimester                                   d. During her postpartum hospitalization
53. Rubella, if acquired by a pregnant patient during the pregnancy may result in congenital defects in
newborn which includes:
1. deaf-mutism                    2. congenital cataract                
                    3.microcephally                 4. congenital heart defect
 
a.       1,2 & 4
b.       2,3& 4
c.       1, 3, & 4
d.       1, 2, 3 & 4     
e.        
54. The following diseases are characterized by the appearance of skin rashes caused by a virus except:
a. measles                                                   c. variola
b. chickenpox                                               d. scarlet fever
 
55. A child with rubella should be isolated from a/an
a. 20-year-old brother living in the same house
b. 3-year-old girl who lives next door
c. 12-year-old sister who had rubeola during childhood
d. 18-year-old female cousin who recently got married
 
 
Situation:  The patient has a sore that does not heal.  Upon examination it reveals leprosy.
56. Mode of transmission of leprosy is
a. Sharing food                                       c. Borrowing utensils
b. Sexual contact                                    d. Prolonged skin to skin contact
57. Late sign of leprosy in male patients is enlargement of the breast known as:
a. Clawing                                            c. Madarosis
b. Lagopthalmos                                    d. Gynecomastia
58. Your role in prevention of leprosy is
                        a. Health Education                               c. Personal Hygiene
b. Proper nutrition                                  d. BCG vaccination
59.  Multi-drug therapy is being implemented by DOH. Where is the initial treatment done?
                        a. Midwife residence                             c. baranggay captain office
b. barangay health center                                   d. rural health unit
 
60. Which of the following is/are mild leprae reaction?
1. Edema                     
2. Sudden increase in the number of lesions
3. Tenderness on the nerve
4. Sudden paralysis
a. 1, 2 & 3 are correct                          
b. 1 & 3 are correct      
c. 2 & 4 are correct
d. 4 only
e. all of the above
 
61.  Nursing care of patients with Diphtheria is provision of complete bed rest. Which nursing
intervention must be done to a child with laryngeal diphtheria?
            a. ice chips every hour
            b. cookies & candies for snacks
            c. Crackers for lunch
            d. small & frequent feeding
 
62. The characteristic lesion of diphtheria is a grayish-white membrane which can block breathing &
swallowing. Which of the following pertains to this membrane?
            a. Koplik’s spots
            b. Forscheimer spots
            c. Papule
            d. Pseudomembrane
63. The bacteria which causes diphtheria is also known as
            a. Spirochete
            b. Klebs-Loeffler bacillus
            c. Cholera
            d. Amoeba
 
Situation: Camille, a 4-year-old girl was brought to a hospital due to paroxysms of cough. Initial
assessment of the physician was pertussis 
64. Which of the following accurately describes symptoms of pertussis?
1. successive coughing
2. vomiting
3. whooping followed by paroxysms of cough
4. barking cough.
      a. 1, 2 and 3                 c. 2 & 4                        e. all of the above
      b. 1 & 3                                    d. 4 only
65. Which laboratory procedure is done to diagnose pertussis?
              a. Saliva examination                                            c. gram staining of nasopharyngeal
secretion
              b. Culture of nasopharyngeal swab             d. blood culture
66. When blood is examined which blood component is elevated in pertussis?
              a. Eosinophils                                            c. RBC
              b. Lymphocytes                                         d. neutrophils
67. A clinic nurse provides instructions to a mother regarding the care of her child who is
diagnosed with croup.  Which statement by the mother indicates a need for further instructions?
  a“ I will place a cool mist humidifier next to my child’s bed.”
b.“Sips of warm fluids during a croup attack will help.”
c.       “ I will give Tynelol for the fever.”
d.      “ I will give cough syrup every night at bedtime.”
Situation: Mr. Ferdinand Lu, a 35-year-old executive is confined in the Medical ward due to pneumonia.
68. After performing your physical assessment  to the patient, which of the following signifies that he is
having pneumonia?
            a. chest auscultation revealing bronchio-vesicular breath sounds
            b. equal chest expansion occurring every inspiration
            c. tactile fremitus is decreased over the affected area
            d. percussion is resonant on the intercostals spaces
69. Steam inhalation was ordered by the physician. The action of this is to
            a. dilate the bronchioles, thus increasing the viscosity of secretions
            b. reduce the viscosity of secretions
            c. increase the viscosity of secretions
            d. inhibit the growth of microorganism
70. Sputum examination for culture & sensitivity was ordered. This would ascertain which of the
following facts?
            a. the correct analgesic to minimize his chest pain during inspiration
            b. the right antibiotic which would be most curative
            c. the right drug to minimize secretions
            d. the correct position during coughing
71. The following are the nursing guidelines for chest physiotherapy. Which of the following should not
be applied to Mr. Lu?
            a. perform postural drainage at least 1 hour after meals
            b. teach deep breathing & coughing exercises
            c. allow patient to continue postural drainage despite feelings of discomfort
            d. avoid chest tapping in areas of severe pain
 
72. a client with pneumonia has a temperature of 102.6 deg F, is diaphoretic, and ahs a
productive cough.  The nurse include which of the following measures in the plan of
care?
a. Position changes every 4 hours
b.      sunctioning to clear secretions
c.       Frequent linen changes
d.      Frequent offering of bed pan
 
73. Bed rest is prescribed for a client with pneumonia during acute phase of the illness. 
Bed rest serves which of the following purposes?
a.       it reduces the cellular demand for oxygen
b.      it decreases the episodes of coughing
c.       it promotes safety
d.      it promotes clearance of secretions
 
74. The cyanosis that accompanies bacterial pneumonia is primarily caused by which of
the following?
a. decreased cardiac output
b.pleural effusion
         c.inadequate peripheral circulation
d.decreased oxygenation of the blood
 
75. A client with pneumonia is experiencing pleuritic chest pain.  Which of the following
describes pleuritic chest pain?
a. a mild but constant aching in the chest
b. severe midsternal pain
c .moderate pain that worsen on inspiration
d .muscle spasm pain that accompanies coughing
 
76.Which of the following measures would most likely be successful in reducing pleuritic
cheast pain in client with pneumonia?
a. encourage the client to breath slowly
b.have the client practice abdominal breathing
c.offer the client incentive spirometry
d.Teach the client to splint the ribcage when coughing
 
77. Which of the following mental status changes may occur when a client with
pneumonia is first experiencing hypoxia?
a. coma
b. apathy
c. Irritability
d. Depression
78. A client with pneumonia has a temperature ranging between 101 and 102 deg F and periods
of diaphoresis.  Based on this information, which of the following nursing interventions would
be priority?
            A.  Maintain Complete bed rest
            b.  administer oxygen therapy
            c.  provide frequent linen changes
            d.  provide fluid intake of 3 liters/day
 
79. Which of the following would be appropriate expected outcome for an elderly client
recovering from bacterial pneumonia?
a.       A respiratory rate of 25 to 30 breaths/ min
b.      The ability to perform activities of daily living
c.       A maximum lost of 5 to 10 pounds of body weight
d.      Chest pain that is minimized by splinting the ribcage.
 
80. A community health nurse is conducting an educational session with community members regarding
tuberculosis. The nurse tells the group that one of the first symptoms associated with tuberculosis is
a.       a bloody, productive cough
b.       a cough with the expectoration of mucoid sputum
c.       chest pain
d.       dyspnea
81. A nurse performs an admission assessment on a client with a diagnosis of tuberculosis. The nurse
reviews the results of which diagnostic test that will confirm this diagnosis?
                        a. bronchoscopy                                                c. sputum culture
                        b. chest x-ray film                                 d. tuberculin skin test
82.  A client who is HIV (+) has had Mantoux skin test. The nurse notes a 7-mm area of induration at the
site of the skin test. The nurse interprets the result as what?
                        a. positive                                             c. inconclusive
                        b. negative                                            d. it needs repeat testing
83.  A nurse in the out-patient department performed Mantoux skin test today (Tuesday) to an adult client.
What statement made by the client indicates that he understood the instruction well?
a.       “I will come back on Saturday for you to read the result.”
b.       “I will come back on Thursday, same time, for you to read the result.”
c.       “I don’t need to come back anymore.”
d.       “I will come back anytime.”
84.  A client exposed to tuberculosis is taking isoniazid (INH) and develops signs & symptoms of the
disease. The client is instructed to add rifampicin & pyrazinamide to the medication regimen. A nurse
explains to the client that the purpose of adding this second medication is
a.       that rifampicin offsets the side effects of isoniazid
b.       to be certain that resistant organisms are eliminated
c.       that these medications potentiate each other
d.       that isoniazid offsets the side effects of rifampicin
85. A nurse is caring for a client diagnosed with tuberculosis. Which assessment, if made by the nurse,
would not be consistent with the usual clinical presentation of tuberculosis and may indicate the
development of a concurrent problem?
                        a. productive cough                               c. chills & night sweats
                        b. anorexia & weight loss                                   d. high-grade fever
86. A nurse has conducted discharge teaching with a client who was diagnosed with tuberculosis. The
client has been taking medication for a week and a half. The nurse evaluates that the client has understood
the information if the client makes which of the following statements?
a.       “I need to continue drug therapy for 2 months.”
b.       “I should not be contagious after at least 2 weeks of medication therapy.”
c.       “I can’t shop at the mall for the next 6 months.”
d.       “I can return to work if a sputum culture comes back negative.”
87. A client has been taking isoniazid (INH) for a month and a half. The client complains to a nurse about
numbness, paresthesias, and tingling in the extremities. The nurse interprets that the client is experiencing
                        a. small blood vessel spasm                    c. impaired peripheral circulation
                        b. hypercalcemia                                               d. peripheral neuritis
88. A client will start a 6-month course therapy with isoniazid (INH). A nurse plans to teach the client to
a.       use alcohol in small amounts only
b.       report yellow eyes or skin immediately
c.       increase intake of cheese & milk
d.       avoid vitamin supplements during therapy
89. A client has been started on long-term therapy with rifampicin. A nurse teaches the client that the
medication
a.       should be double dosed if one dose is forgotten
b.       may be discontinued independently if symptoms are gone in 3 months
c.       should always be taken with food or antacids
d.       causes orange discoloration of sweat, tears, urine, & feces
90. A nurse has given a client taking ethambutol information about the medication. The nurse determines
that the client understands the instructions if the client states to immediately report
a.       gastrointestinal side effects
b.       impaired sense of hearing
c.       orange-red discoloration of body secretions
d.       difficulty in discriminating the color red from green
Situation: Tuberculosis, a primary respiratory disease is common among malnourished individuals living
in crowded places. The DOH survey last 1997 revealed that there are approximately 16 million afflicted
with the disease.
91. The bacteria causing the disease is
            a. Mycobacterium bovis
            b. Mycobacterium tuberculosis
            c. Mycobacterium africanum
            d. Mycobacterium leprae
Situation:  Mang Romy is malnourished living in a squatter’s area.  He came in the health center due to
loss of weight easy, fatigability, chronic cough and low grade fever.
92. The main source of infection pulmonary tuberculosis is:
a. crowded living quarters
b. unavailability of potable drinking water
c.  direct contact with infected person
d. unsanitary surroundings
93. A physician orders an X-ray examination for patient.  What is the purpose for this?
a. To check if patient have relatives with TB
b. To see the evidence of having the disease
c. Determine severity of the lesions in the lungs
d. To aid in doctors prescriptions.
94. The nurse notes 12 mm of induration at the site of a Mantoux test when a client returns to the health
office to have it read. The nurse should explain to the client that this is
            a. Test result is negative, & no follow-up is needed.
            b. Test was used for screening and a TINE test will now be given.
            c. Skin test is inconclusive and will have to be repeated in six weeks.
            d. The result is positive and a need for further tests, including chest x-ray is indicated
95. Staff nurses learn that a patient they have been caring for during the last few weeks has just been
diagnosed with tuberculosis. When the nurses express concern about contracting tuberculosis themselves,
the charge nurse’s response should be based on which of the following statements?
            a. Tuberculosis is easily treated with short course of antibiotics
            b. The Mantoux test is used to confirm diagnosis of tuberculosis
            c. Tuberculosis is not highly infectious when standard precautions are followed
            d. Vaccination with BCG will be used to immunize the nurses against infection
96. Before returning a child who is being treated for tuberculosis to his home, the community health nurse
determines
            a. that the child has a private room
            b. that all family members have been tested
            c. proper room ventilation
            d. home school placement
97. Which of the following statements, if made by a patient who has tuberculosis, would indicate a
correct understanding of disease transmission?
            a. “The disease can be acquired from breast milk.”
            b. “The disease does not spread beyond the lungs.”
            c. “I contracted the disease through bird droppings.”
            d. “I can be infected more than once with this disease.”
98. Which of the following instructions should a nurse give to a patient who has tuberculosis and is
receiving rifampicin?
            a. “Stay out of the sun while you are taking the medication.”
            b. “Expect your urine to turn reddish-orange.”
            c. “Take your medication on an empty stomach.”
            d. “Take your medication until your skin test is negative.”
99. A patient who is taking INH, Rifampicin, PZA and pyridoxine for the treatment of tuberculosis asks a
nurse why the Vitamin B6 (pyridoxine) is necessary. The nurse would respond that
            a. “Vitamin B6 is necessary for the absorption of INH.”
            b. “Vitamin B6 activates the metabolism of INH.”
            c. “INH leads to Vitamin B6 depletion, which leads to neurotoxic effects.”
            d. “INH can cause anorexia, which leads to vitamin B6 depletion.”
100. What is the rationale that supports multidrug treatment for clients with tuberculosis?
a.       Multiple drugs potentiate the drug actions
b.      Multiple drugs reduce undesireable side effects
c.       Multiple drugs allow reduced dosages to be given
d.      Multiple drugs reduce development of resistant strains of the bacteria
 
101. The client with TB is to be discharged home with community health nurse follow-up.  Of
the following interventions, which would have the highest priority?
a.       offering the client emotional support
b.      teaching the client about the disease and its treatment
c.       coordinating various agency services
d.      Assessing the client’s environment for sanitation
 
102. The nurse should caution sexually active female clients taking INH that the drug has which
of the following effects?
a.       Increases the risk of vaginal infection
   b. Has mutagenic effects on ova
c. Decreases the effectiveness of oral contraceptives
d. Inhibits ovulation
 
103.Clients who have had active TB are at risk for recurrence.  Which of the following
conditions increases that risk?
b.      Cool and damp weather
c.       Active exercise and exertion
d.      Physical and emotional stress
e.       Rest and inactivity
 
104.The nurse should include which of the following instructions when developing a teaching
plan for clients who are receiving INH and rifampicin for treatment of TB?
a. Take the medications with antacids
b. Double the dosage if a drug dose is forgotten
c. Increase intake of dairy products
d. Limit alcohol intake
 
Situation:  Mandy started to complain severe headache and high grade fever for almost two weeks.  She
remembered eating “sapin sapin” in a nearby store.  The initial impression of the admitting doctor was
typhoid fever.
105. The organ of predilection of typhoid infection is:
a. small intestine                                           c. gallbladder
b. colon                                                       d. Spleen
106.  Mandy reported presence of black or tarry stool a day prior to admission.  Based on the natural
history of disease, she is on what stage of the disease process?
a. prodomal satge                                         c. defervescence stage
b. fastidial stage                                           d. recovery stage
107. Nurse Arianne examined Mandy.  She was trying to look for the an exanthema of typhoid fever
which is
a. maculopapular rashes on trunk                  c. bluish discoloration on arms
b. petechial rashes on extremities                  d. migratory painless nodules
108. The confirmatory test for typhoid on the first  week of illness is
a. Widal test                                                            c. Typhidot
b. Urine culture                                            d. Blood culture
109. The stool culture of a child with profuse diarrhea reveals Salmonella bacilli.  After teaching
the mother about the course of Salmonella enteritis, which of the following statements by the
mother indicates effective teaching?
a.“Some people become carriers and stay infectious for a long time”
   b.“ After the acute stage passes the organism is usually not present in the stool.”
   c.“ Although the organism may be alive indefinitely, in time it will be of
no danger to anyone.”
   d.“ If my child continues to have the organism in the stool, an antitoxin can help
        destroy the organism.”
 
110. Which of the following would be the best activity for the nurse to include in the plan of care
for an infant experiencing severe diarrhea?
a.       Monitoring the total 8 hour formula intake
b.      Weighing the infant each day
c.       Checking the anterior fontanel every shift
d.      Monitoring abdominal skin turgor every shift
 
111. Which of the following would be most appropriate for the nurse to teach the mother of a 6
month old infant hospitalized with severe diarrhea to help comfort her infant who is fussy?
 
a. Offering a pacifier
b.Placing a mobile above the crib
c. Sitting at the crib side talking to the infant
d.Turning the television on to cartoons
 
112.Which of the following nursing diagnosis would be appropriate for the nurse to identify as a
priority diagnosis for an infant just admitted to the hospital with a diagnosis of gastroenteritis?
a.Pain related to repeated episodes of vomiting
b.Deficient Fluid volume related to excessive losses from severe diarrhea
c.Impaired parenting related to infant’s loss of fluid
d.Impaired Urinary Elimination related to increased fluid intake feeding
pattern
 
113.Which of the following would the nurse use to determine achievement of the expected
outcome for an infant with severe diarrhea and nursing diagnosis of Deficient Fluid volume
related to passage of profuse amounts of watery diarrhea?
a. Moist mucous membranes
b.Passage of a soft, formed stool
            c.Absence of diarrhea for a 4 hour period
d.Ability to tolerate intravenous fluids well
 
114.A child is admitted to the pediatric unit with the diagnosis of severe gastroenteritis.  Which
of the following would be most appropriate for the nurse to do?
a. Institute standard precautions
b. Place the child in a semiprivate room
c. Use regular eating utensils
d. Single- bag all linens
 
115. Of the following would most likely alert the nurse to the possibility that a preschooler is
experiencing moderate dehydration?
a. Vomiting
b. Diaphoresis
c. Absence of tear formation
d. Decreased urine specific gravity
 
116.The physician orders IV fluid replacement therapy with potassium chloride to be added for a
child with severe gastroenteritis.  Before adding the potassium chloride to the IV, which of the
following assessments would be most important?
a. Ability to void
b. Passage of stool today
c. Baseline echocardiogram
d. Serum calcium level
 
117.Which of the following would first alert the nurse to suspect that a child with severe
gastroenteritis who has been receiving IV therapy for the past several hours may be developing
circulatory overload?
a. a drop in blood pressure
b. change to slow, deep respirations
c. auscultation of moist crackles
d. marked increase in urine output
118. To help prevent an infant from contracting infectious diarrhea, the nurse should instruct the mother
on which topic?
            a. When to introduce solid foods into the infant’s diet
            b. How to prepare, handle & store infant formula
            c. When to have the child immunized
            d. When to bring the infant to the clinic for routine check-ups
119. A nursing student is assigned to admit to the hospital a child who has been experiencing vomiting &
diarrhea. A physician establishes a diagnosis of acute gastroenteritis & isotonic dehydration. A nursing
instructor asks the student to describe isotonic dehydration. The nursing student responds accurately by
telling to the instructor that isotonic dehydration
a.       occurs when water & electrolytes are lost in about the same proportions as they
exist in the body
b.       occurs when the loss of electrolytes is greater than the loss of water
c.       occurs when the loss of water is greater than the loss of electrolytes
d.       causes the serum sodium level to rise above 150 mEq/L
120. A clinic nurse is assessing a child for dehydration. The nurse determines that the child is moderately
dehydrated if which symptom is noted on assessment?
                        a. flat fontanels                                     c. moist mucous membranes
                        b. pale skin color                                               d. oliguria
121. Bloody & mucoid stool associated with tenesmus is pathognomonic of
                        a. diphtheria                                          c. bacillary dysentery
                        b. violent dysentery                                d. amoebic dysentery
 
122. The intermediate vector of schistosomiasis is
a. mosquitoes                                                           c. rats
b. flies                                                                     d. fresh water snail
 
123. Diet of patient with acute parotitis should contain
            a. High Na, High CHO, High fats
            b. Low Na, Low CHO, Low fats
            c. High CHO, High fats
            d. High CHO, Low fats
 
124.A mother is worried because her 5-year-old son acquired mumps infection. She asks about the most
severe complication of this infection. The best response of the nurse should be
a.       “You don’t have to worry because there is no known complication of mumps.”
b.       “Although rare, the most severe complication of mumps is encephalitis.”
c.       “Your son will become sterile for life.”
d.       “Your child may develop pneumonia.”
 
Situation: Mr. Reyes,  25-year-old clerk is admitted to the hospital with a chief complaint of easy
fatigability associated with fever, amber-colored urine & clay colored stools. He is diagnosed with Acute
Viral Hepatitis type B. On the fourth hospital day, he developed jaundice & strongly insisted that his
visitors be restricted to her immediate family members.
125. Potential problems that a nurse may encounter include all of the following except
            a. Bleeding
            b. Pruritus
            c. Weight loss
            d. Hyperglycemia
126 Based on the data given, one of the nursing diagnosis is
            a. Depression related to feelings of guilt
            b. Anxiety related to feeling of impending doom
            c. Body image disturbance related to altered skin appearance
            d. Anger & hostility related to restriction of physical activity
127. The nurse observes that Mr. Reyes has clay-colored stools. He asked you why is this so. The reason
for this is that
            a. Hepatic uptake of bilirubin is impaired
            b. Excretion of fecal urobilinogen is increased
            c. Conjugated bilirubin reenters the bloodstream
            d. Excretion of conjugated bilirubin into the intestines is decreased
128. To best reduce the risk of spread of Mr. Reyes’ disease, the nurse must observe the proper disposal
of the patient’s
            a. Urine & feces
            b. Nasogastric secretions
            c. Used needles & syringes
            d. Feces & oral secretions
 
129.The nurse plans care for the client with Hepatitis A with the understanding that the causative
virus will be excreted from the client’s body primarily through the:
a. skin
b. feces
c. urine
d. blood
 
130. The nurse is planning a staff development program for health care staff on how to care for
the clients with hepatitis A.  Which of the following precautions would the nurse indicate as
essential when caring for clients with Hepatitis A?
a. Gowning when entering aclient’s room
b. Wearing a mask when providing care
c. Assigning the client to a private room
d. Wearing gloves when giving direct care
 
131. When developing a plan of care for the client with viral hepatitis, the nurse should
incorporate nursing orders that reflect the primary treatment.  Emphasis will be on ensuring that
the client receives which of the following?
a. adequate bed rest
b. generous fluid intake
c. regular antibiotic therapy
d. daily IV theraphy
 
132.In a client with viral hepatitis, the nurse would closely assesses for indicators of which of the
following abnormal laboratory values?
a. prolonged PT
b.decreased blood glucose level
c.elevated serum potassium
d.decreased serum calcium
 
133. Which of the follwing diets would most likely be prescribed for a client with viral hepatitis?
a. high fat, low protein
b. high protein, low carbohydrates
c.high crbohydrates, high calorie
d. low sodium, low fat
134. The nurse develops a teaching plan for the client about how to prevent the transmission of
hepatitis A.  Which of the following discharge instructions is appropriate for the client?
a. Spray the house to eliminate infected insects
b. Tell the family members to try to stay away from the client.
c. Tell the family members to wash their hands frequently
d.  Disinfect all clothing and eating utensils
 
135. The nurse assesses that the client with hepatitis is experiencing fatigue, weakness, and a
general feeling of malaise.  The client tires rapidly during morning care .  Based  on this
information, which of the following would be an appropriate nursing diagnosis?
a. Impaired physical mobility related to malaise
b. Self-cared Deficit related to fatigue
c. Ineffective Coping related to long term illness
d. Activity Intolerance related to fatigue
 
136. A client has been admitted to the hospital with a diagnosis of hepatitis B.  the client tells the
nurse, “ Ifeel so isolated from my friends and family.  Nobody wants to be around me.”  What
would be the most appropriate nursing diagnosis for this client?
a. Anxiety related to feelings of isolation
b. Social isolation related to significant other’s fear of contracting disease
c. Powerlessness related to lack of social support
d. Low-self Esteem related to feelings of rejection.
137. A delivery room nurse was unable to perform newborn care to a neonate. In the history of the
postpartum mother, she recently was diagnosed with gonococcal infection that was untreated. Which of
the following will most likely to occur in the neonate?
a. The neonate will have bleeding problems due to lack of vitamin K
b. The neonate will develop ophthalmia neonatorum
c. The neonate will not have any medical problems
d. The neonate will develop respiratory failure
138. Cristina Yap, a 30-year-old female came to a clinic due to burning sensation in the genital area. The
physician diagnosed her with non-gonococcal urethritis. The nurse knows that the most likely etiologic
agent of non-gonococcal urethritis is
a. Neiserria gonorrhea                               c. Chlamydia trachomatis
b.Trichomonas vaginalis                            d. Haemophilus ducreyi
 
Situation:  A 24-year-old male presents to the OPD with a 3 day history of burning sensation on urination
which progressed to the development of purulent, yellowish-to-greenish urethral discharge.  The nurse
suspects of STD.
139. On laboratory examination, the diagnosis of gonorrhea is confirmed by which of the following on
the report of the staining result?
a. Gram negative intracellular diplocci                        c. Numerous spirochetes
b. Gram positive extracellular diplococci        d. Gram negative extracellular diplococci
 
 
140. The nurse knows that the easiest factor to break in the infectious process to prevent STD spread
would be
a. Reservior                                                 c. Agent
b. Mode of transmission                                d. Portal of entry
141. The prevention and control of STD is made difficult by
a. Lack of drugs specific for STD                 c. No government program on STD
b. Self-medication of infected persons                       d. Lack of  better laboratory facility
142. Nurse is very active in contact tracing and epidemiologic treatment of all gonococcal identified
contacts to prevent
a. Development of resistant strain                 c. Non venereal transmission
b. Asymptomatic infection                            d. Re-infection
143. The patient made the following statement to the nurse.  Which statement would need reinforcement
of health teachings?
a. I need to take drugs for prophylaxis the next time I do it
b. Gonorrhea does not confer lifelong immunity
c. The disease can also be transmitted through other means
d. I will finish the prescribed medication to prevent resistance of microorganism
 
144. A female client with gonorrhea informs the nurse that she had sexual intercourse with her boyfriend
and asks the nurse “Would he experience any symptoms? The nurse responds that in males, the initial
manifestation is
            a. Impotence
            b. Scrotal swelling
            c. Sterility
            d. Dysuria
Situation: Mr. Larry Hipolito, a 25 y/o male came to an out-patient clinic complaining of pain over the
hypogastrium & yellowish discharge from his urethra. A diagnosis of gonorrhea is made.
145. Since Larry has gonorrhea, he most likely has which of these symptoms?
            a. a maculo-papular rash on the pubic area
            b. a painless ulcer on the penis
            c. wart-like lesions on the scrotum
            d. pain upon urination
146. Larry was instructed to remain in the clinic for at least 30 minutes so that adverse reactions to the
medication can be detected. Which of these comments made by Larry is indicative of an adverse reaction
to the specific drug?
            a. “My chest feels tight”
            b. “I need to go to the bathroom.”
            c. “I need a glass of water.”
            d. “My hands are shaky.”
147. Larry asked for possible complications if he had not received treatment for gonorrhea. He would be
told that one such complication is
            a. liver atrophy
            b. epididymis
c. peripheral vascular disease
d.  ectopic pregnancy
148. Characteristic lesion of primary syphilis is
            a. chancroid
            b. condyloma lata
            c. gumma
            d. chancre
149. A client is suspected of having late-stage syphilis. When obtaining a health history, the nurse
recognizes that the statement by the client that would most support this diagnosis would be
            a. “I noticed a wart on my penis.”
            b. “I have sores all over my mouth.”
            c. “I have been losing a lot of hair lately.”
            d. “I’m having trouble keeping my balance.”
Situation: Mark Smith, a 37 y/o male has 19 years history of IV drug abuse. He has poor appetite, nausea,
& weight loss of 10 kg in the past 6 months. A diagnosis of HIV was made.
150. To initiate a nursing care plan for Mr. Smith, the nurse should consider that a depressed immune
system is caused by which of these pathophysiological alterations?
            a. Decreased leucocyte count
            b. low platelet count
            c. reduction of  nucleated RBC
            d. Increased sedimentation rate
151. Lesions that will characterize Mr. Smith is in the advanced stage of the illness is
            a. condyloma lata
            b. gumma
            c. Kaposis sarcoma
            d. Pneumocyctis carinii pneumonia
 
152. A patient who is HIV positive is admitted to an isolation unit for treatment of tuberculosis. The nurse
observes that the patient has difficulty following instructions, short-term memory loss & difficulty
concentrating. The nurse should be aware that these behaviors
            a. will disappear when isolation is no longer required
            b. are symptoms of AIDS dementia complex
            c. are normal for the early stage of infection
            d. are a result of mycobacterium infection
153. A patient who has AIDS develops Pneumocyctis carinii pneumonia (PCP). The patient asks the
nurse, “How did I get this pneumonia?” The nurse’s response should be based on which of these
statements about PCP?
            a. It occurs in immunosuppresed persons from proliferation of organisms that are
            normally present  in the body
            b. It is transmitted from close contact with an infected individual who has a suppressed
            immune system.
            c. It results from exposure to a carrier of the organism who has not taken appropriate
            precautions.
            d. It is most often acquired from unprotected sex with an infected individual.
154. An HIV-positive patient is prescribed zidovudine and ganciclovir. The nurse should be aware that
both drugs have which of the following side effects?
            a. Peripheral neuropathy
            b. Insomnia
            c. Anticholinergic effects
            d. Bone marrow suppression
155. Blood screening tests of the immune system of a client with AIDS would indicate
            a. a decrease in CD4 T cells
            b. an increase in thymosine (thymic hormone)
            c. an increase in immunoglobulin E
            d. a decrease in erythrocytes
156. Mr. Santos, a 48-year-old male is an HIV suspect admitted in San Lazaro. In assessing the patient’s
condition for possible HIV infection, the least expected symptom to occur is
            a. Night sweating
            b. Fever
            c. Fungal infection in the mouth
            d. Acute diarrhea
157. The nurse is caring for the client with acquired immunodeficiency syndrome. The nurse detects early
infection with Pneumocystis carinii by monitoring the client for which clinical manifestation?
a.       dyspnea on exertion                         c. fever
b.       dyspnea at rest                                d. cough
158. The nurse is caring for the client with AIDS who is experiencing night fever & sweats. Which
nursing intervention would be the least helpful in managing this symptom?
a.       keep a change of bed linens nearby in case they are needed
b.       administer an antipyretic after the client has spike of fever
c.       make sure the pillow has a plastic cover
d.       keep liquids at bedside
159. The client exposed to human immunodeficiency virus about 3 months ago has seroconverted to an
HIV-positive status. The nurse anticipates that the client will experience which of the following at this
time?
                        a. oral lesions                                        c. purplish skin lesions
                        b. chronic cough                                                d. no signs & symptoms
160. The client with AIDS has raised dark purplish-colored lesions on the trunk of the body. The nurse
anticipates that which of the following procedures will be done to confirm whether these lesions are due
to Kaposi’s sarcoma?
                        a. Enzyme-linked immunosorbent assay  c. western blot
                        b. skin biopsy                                        d. lung biopsy
 
 
 
 
 
 
@@@@@@@@@@@@@@@@@@@@@@@@@@@@@@@@@@@@@@@@@@
@@@@@@@@@
 
 
CD COMPRE EXAM 2
QUESTIONS
 
1. A nurse is performing an assessment of a prenatal client being seen in the clinic for the
first time.  Following the assessment, the nurse determines that which piece of data places
the client into the high risk category for contracting human immunodeficiency virus
(HIV)?
a. living in the area where population rate of HIV infection is low.
b. A history of IV drug use in the past year
c. History of one sexual partner within the past 10 years
d. A spouse who is heterosexual and had only one sexual partner in the past 10
years.
 
2. A client with AIDS gets recurrent Candida infections (thrush) of the mouth.  The nurse
has given instructions to the client to minimize the occurrence of thrush and determines
that the client understands the instructions if which statement is made by the client?
a. “ I should brush my teeth and rinse my mouth once a day.”
b. “ I should use a strong mouthwash at least once a week.”
c. “Increasing red meat in my diet will keep this from recurring.”
d. “I should use warm saline or water to rinse my mouth.”
 
3. A nurse is teaching a client with AIDS how to avoid food borne illness.  The nurse
instructs the client to prevent acquiring infection from food by avoiding which of the
following items?
a. raw oysters
b. pasteurized milk
c. products with sorbitol
d. bottled water
 
4. A client with histoplasmosis has an order for ketoconazole (Nizoral).  The nurse teaches
the client to do which of the following while taking this medication?
a. Take the medication on an empty stomach
b. Take the medication with an antacid
c. Avoid exposure to sunlight
d. Limit alcohol to 2  ounces per day
 
5. A nursing instructor asks a nursing student to describe live or attenuated vaccines.  The
student tells the instructor that these types of vaccines are:
a. vaccines that have their virulence (potency) diminished so as to not produce a full-
blown clinical illness.
b.  vaccines that contain pathogens made inactive by either chemical or heat.
c.  bacterial toxins that have been made inactive by either chemicals or heat.
d.  vaccines that have been obtained from the pooled blood of many people and provide
antibodies to a variety of diseases.
 
6. A nurse is performing an assessment on a 3 year old child with chicken pox.  The child’d
mother tells the nurse that the child keeps scratching at night, and the nurse teaches the
mother about measures that will prevent an alteration in skin integrity.  Which statement
by the mother indicates that teaching was effective?
a. “ I will apply generous amounts of a cortisone cream to prevent itching.”
b. “ I need to place white gloves on my child’s hands at night.”
c. “I need to keep my child ia a warm room at night so that covers will not cause my
child to scratch.”
d. “ I will give my child a glass of warm milk at bedtime to help my child sleep.”
 
7. A nurse instructs a client with hepatitis about measures to control fatigue.  The nurse
determines that the client needs additional instructions if the client states to:
a. plan rest periods after meals
b. rest between activities
c. perform personal hygiene if not fatigue
d. complete all daily activities in the morning when the client is most rested
 
8. A clinic nurse has provided home care instructions to a female client who has been
diagnosed with recurrent trichomoniasis.  Which statement by the client indicates a need
for further instructions?
a. “ I need to perform good perineal hygiene.”
b. “ I need to refrain from sexual intercourse.”
c. “ I need to discontinue treatment if my menstrual cycle begin.”
d. “ I need to take metronidazole (Flagyl) for seven days.”
 
9. A client with AIDS has a nursing diagnosis of Imbalanced Nutrition: Less Than Body
Requirements.  The nurse has instructed the client  about methods to maintain and
increase weight.  The nurse determines that the client would benefit  from further
instruction if the client stated to:
a. Eat low-calorie snacks between meals
b. Eat small, frequent meals throughout the day
c. Consume nutrient-dense foods and beverages
d. Keep easy-to-prepare foods available in the home
 
10. A nurse is teaching a client with histoplasmosis infection about prevention of future
exposure to infectious sources.  The nurse determines that the client needs further
instructions if the client states that the potential infectious sources include:
a. grape arbors
b. mushroom cellars
c. floors of chicken houses
d. bird droppings
 
11. A clinic nurse provides instructions to a mother regarding the care of her child who is
diagnosed with croup.  Which statement by the mother indicates a need for further
instructions?
a. “ I will place a cool mist humidifier next to my child’s bed.”
b. “Sips of warm fluids during a croup attack will help.”
c. “ I will give Tynelol for the fever.”
d. “ I will give cough syrup every night at bedtime.”
 
12. A client with Chlamydia infection has received instructions on self-care and prevention
of further infection.  The nurse determines that the client needs reinforcement of the
instructions if the client states to:
a. Reduce the chance of reinfection by limiting the number of sexual partners
b. Use latex condoms to prevent disease transmission
c. Return to the clinic as requested for follow-up culture in 1 week.
d. Use antibiotics prophylactically to prevent symptoms of Chlamydia
 
 
13. A child is seen in the health care clininc and initial testing for HIV is performed because
of the child’s exposure to HIV infection.  Which home care instruction would the nurse
provide to the parents of the child?
a. Avoid all immunization until the diagnosis is established
b. Avoid sharing toothbrushes
c. Wipe up any blood spills with soap and water and allow to air dry
d. Wash hands with half-strength bleach if they come in contact with the child’s
blood
 
14. A nurse is caring for a client with AIDS.  Which finding noted in the client indicates the
presence of an opportunistic respiratory infection?
a. white plaques located on the oral mucosa
b. fever, exertional dypnea, and nonproductive cough
c. loss of sight
d. ulcerated perirectal lesions
 
15. A nurse is caring for a child diagnosed with rubeola.  The nurse notes that the physician
has documented the presence of Koplik spots.  Based on this documentation, which of the
following would the nurse expect to note on assessment of the child.
a. petechiae spots that are reddish and pinpoint on the soft palate.
b. Whitish vesicles located across the chest
c. Small, blue-white spots with a red base found on the buccal mucosa
d. Pinpoint petechiae noted on both legs
 
16. A mother brings the child to the health care clinic.  The child has been complaining of
severe headaches and has been vomiting.  The child has a high fever, and the nurse notes
the presence of nuchal rigidity in the child.  The nurse suspects a possible diagnosis of
bacterial meningitis. The nurse continues to assess the child for the presence of Kernig’s
sign.  Which finding would indicate the presence of this sign.
a. Inaability of the child to extend the legs fully when lying  supine
b. Flexion of the hips when the neck is flexed from a lying position
c. Pain when the chin is pulled down to the chest
d. Calf pain when the foot is dorsiflexed
17. A nurse determines that a Mantoux tuberculin skin test is positive.  In order to most
accurately diagnose TB, the nurse plans to consult with the physician to follow-up the
skin test with a:
a. Chest X-RAY
b. CT scan of the chest
c. Sputum culture
d. CBC
 
18. A nurse is planning care for a child with an infectious and communicable disease.  The
nurse determines that the primary goal is that the:
a. Child will experience only minor complication
b. Child will not spread the infection to others
c. Public health department will be notified
d. Child will experience mild discomfort
 
19. A mother of a child with mumps calls the health care clinic to tell the nurse that the child
has been lethargic and vomiting.  The nurse most appropriately tells the mother:
a. to continue to monitor the child
b. that lethargy and vomiting are normal manifestation of mumps
c. to bring the child to the clinic to be seen by the physician
d. that as long as there is no fever, there is nothing to be concerned about
 
20. A child with a diagnosis of hepatitis B is being cared for at home.  The mother of the
child calls the health care clinic and tells the nurse that the jaundice seems to be
worsening.  The nurse makes which response to the mother?
a. “ The hepatitis may be spreading.”
b. “ You need to bring the child to  the health care clinic to see the physician.”
c. “The jaundice may appear to get worse before it resolves.”
d. “ It is necessary to isolate the child from the others.”
 
21. A nurse is planning to teach a client who is newly diagnose with TB on how to prevent
the spread of TB.  Which instruction would be least effective in preventing the spread of
TB
a. Teach the client to cover the mouth when coughing
b. Teach the client to sterilize dishes at home
c. Teach the client to properly dispose of facial tissues
d. Teach the client that close contacts should be tested for TB
 
22. A nurse performs an initial assessment on a pregnant client and determines that the client
is at risk for toxoplasmosis.  The nurse would teach the client which of the following to
prevent exposure to this disease?
a. Wash hands only before meals
b. Eat raw meats
c. Avoid exposure to litter boxes used by cats.
d. Use tropical corticosteroids treatments prophylactically.
 
 
23. A home care nurse begins caring for a 25 year old female client who has just been
diagnosed with HIV infection.  The client asks the nurse, “ How could this have
happened?”  The nurse responds to the question based on the most frequent mode of HIV
transmission, which is
a. hugging HIV positive sexual partner without using barrier precautions.
b. Inhaling cocaine
c. Sharing food utensils with an HIV positive person without proper cleaning of the
utensils
d. Having sexual intercourse with an HIV positive person without using a condom
 
24. A client with HIV is taking zidovudine (AZT). AZT is a drug that acts to:
a. destroy the virus
b. enhance the body’s antibody production
c. slow replication of the virus
d. neutralize the toxins produced by the virus
 
25. Women who have human papillomavirus (HPV) are at risk for development of:
a. sterility
b. cervical cancer
c. uterine fibroid tissue
d. irregular menses
 
26. Which of the following nursing diagnosis would most likely be a priority for a client with
herpes genitalis ?
a. Disturbed sleep pattern
b. Imbalance Nutrition: Less than Body Requirements
c. Pain
d. Ineffective Breathing Pattern
 
27. The primary reason that a herpes simplex virus (HSV) infection is a serious concern to a
client with HIV infection is that it
a. is an acquired immunodeficiency virus –defining illness
b. is curable only after 1 year of antiviral therapy
c. leads to cervical cancer
d. causes severe electrolyte imbalances
 
28. In Educating a client about HIV, the nurses should take into account the fact that the most
effective method known to control the spread of HIV infection is:
a. premarital serological screening
b. prophylactic treatment of exposed people
c. laboratory screening of pregnant woman
d. ongoing sex education about preventive behaviors
29. A male client with HIV infection becomes depressed and tells the nurse,” I have nothing
worth living for now.” Which of the following statements would be the best response by
the nurse?
a. “ You are yopung person and have a great deal to live for.”
b. “ You should not be too depressed; we are close to finding cure for AIDS”
c. “ You are right; it is very depressing to have HIV”
d. “ Tell me more about how you are feeling about being HIV positive.”
 
30. The organism responsible for causing syphilis is classified as a:
a. virus
b. fungus
c. rickettsia
d. spirochete
31. The typical chancre of syphilis appears as
a. a grouping of small, tender pimples
b. an elevated wart
c. a painless, moist chancre
d. an itching, crusted area.
 
32. When interviewing a client with newly diagnosed syphilis, The public health nurse
should be aware that the spread of the disease ca be controlled by
a. motivating the client to undergo treatment
b. obtaining a list of the client’s sexual contacts
c. increasing the client’s knowledge of the disease
d. reassuring the client that records are confidential.
 
33. Benzathine penicillin G, 2.4 million units IM, is prescribed as treatment for an adult
client with primary syphilis.  The intramuscular injection is administered in
a. the deltoid
b. the upper outer quadrant of the buttock
c. the quadriceps lateralis of the thigh
d. the midlateral aspect of the thigh
 
34. A priority nursing diagnosis for a client with primary syphilis
a. Deficient Knowledge relataed to lack of exposure to information about mode of
transmission
b. Pain related tocutaneous skin lesion on palms and soles
c. Ineffective tissue perfusion related to bleeding chancre
d. Disturbed body image related to alopecia
 
35. An 18 year old female college student is seen at the university health center.  She
undergoes a pelvic examination and is diagnosed with gonorrhea.  Which of the
following responses by the nurse would best when the client states that she is nervous
about upcoming pelvic examination?
a. “ Can you tell me more about how you are feeling”
b. “ You’re not alone.  Most women feel uncomfortable about this examination”
c. “ Do not worry about Dr. Smith.  He is specialist in female problems.”
d. “ We’ll do everything we can to avoid embarrassing you.”
 
36. When educating a female with gonorrhea, the nurse should emphasize that for women,
gonorrhea
a. is often marked by symptoms of dysuria or vaginal bleeding
b. does not lead to serious complication
c. can be treated but not cured
d. may not cause symptoms until serious complication occurs
 
37. Which of the following groups has experienced the greatest rise in the incidence of STDs
over the past two decades
a. teenagers
b. divorced people
c. young married couples
d. older adults
 
38. A female client with gonorrhea informs the nurse that she has had sexual intercourse with
her boyfriend and asks the nurse, “  Would he have any symptoms?”  The nurse responds
that in men the symptoms of gonorrhea include
a. impotence
b. scrotal swelling
c. urinary retention
d. dysuria
 
39. The nurse assesses the mouth and oral cavity of a client with HIV because the most
common opportunistic infection initially presents as
a. HSV lesions on the lips
b. Oral candidiasis
c. CMV infection
d. Aphthae on the gingival
 
40. The nurse is planning a community education program on how to prevent the
transmission of viral hepatitis.  Which of the following types of hepatitis is considered to
be primarily a sexually transmitted disease?
a. Hepatitis A
b. Hepatitis B
c. Hepatitis C
d. Hepatitis D
 
41. The nurse would expect the client to exhibit which of the following symptoms during
icteric phase of viral hepatitis?
a. Tarry stool
b. Yellowed sclera
c. Shortness of breath
d. Light, frothy urine
 
42. The nurse plans care for the client with Hepatitis A with the understanding that the
causative virus will be excreted from the client’s body primarily through the:
a. skin
b. feces
c. urine
d. blood
 
43. The nurse is planning a staff development program for health care staff on how to care
for the clients with hepatitis A.  Which of the following precautions would the nurse
indicate as essential when caring for clients with Hepatitis A?
a. Gowning when entering aclient’s room
b. Wearing a mask when providing care
c. Assigning the client to a private room
d. Wearing gloves when giving direct care
 
44. When developing a plan of care for the client with viral hepatitis, the nurse should
incorporate nursing orders that reflect the primary treatment.  Emphasis will be on
ensuring that the client receives which of the following?
a. adequate bed rest
b. generous fluid intake
c. regular antibiotic therapy
d. daily IV theraphy
 
45. Which of the following test results would the nurse use to assess the liver function of a
client with viral hepatitis?
a. glucose tolerance
b. creatinine clearance
c. serum transaminase
d. serum electrolytes
 
46. In a client with viral hepatitis, the nurse would closely assesses for indicators of which of
the following abnormal laboratory values?
a. prolonged PT
b. decreased blood glucose level
c. elevated serum potassium
d. decreased serum calcium
 
47. Which of the follwing diets would most likely be prescribed for a client with viral
hepatitis?
a. high fat, low protein
b. high protein, low carbohydrates
c. high crbohydrates, high calorie
d. low sodium, low fat
48. The nurse develops a teaching plan for the client about how to prevent the transmission
of hepatitis A.  Which of the following discharge instructions is appropriate for the
client?
a. Spray the house to eliminate infected insects
b. Tell the family members to try to stay away from the client.
c. Tell the family members to wash their hands frequently
d. Disinfect all clothing and eating utensils
 
49. The nurse assesses that the client with hepatitis is experiencing fatigue, weakness, and a
general feeling of malaise.  The client tires rapidly during morning care .  Based  on this
information, which of the following would be an appropriate nursing diagnosis?
a. Impaired physical mobility related to malaise
b. Self-cared Deficit related to fatigue
c. Ineffective Coping related to long term illness
d. Activity Intolerance related to fatigue
 
50. A client has been admitted to the hospital with a diagnosis of hepatitis B.  the client tells
the nurse, “ Ifeel so isolated from my friends and family.  Nobody wants to be around
me.”  What would be the most appropriate nursing diagnosis for this client?
a. Anxiety related to feelings of isolation
b. Social isolation related to significant other’s fear of contracting disease
c. Powerlessness related to lack of social support
d. Low-self Esteem related to feelings of rejection.
 
51. A 79 year old female client is admitted to the hospital with the diagnosis of bacterial
pneumonia.  While obtaining the client’s health history, the nurse learns that the client
has osteoarthritis, follows a vegetarian diet, and is very concerned with cleanliness. 
Which of the following would most likely be a predisposing factor for the diagnosis of
pneumonia?
A.     Age
B.     Osteoarthritis
c.       Vegetarian diet
d.      Daily bathing
 
52. A client with bacterial pneumonia is to be started on IV antibiotics.  Which of the
following diagnostic tests must be completed before antibiotic therapy begins?
a. Urinalysis
b. Sputum culture
c. Chest radiograph
d. Red blood cell count
 
53. When caring for the client who is receiving an aninoglycoside antibiotic, the nurse
monitors which of the following laboratory values?
a. Serum sodium
b. serum potassium
c. serum creatinine
d. serum calcium
 
54. a client with pneumonia has a temperature of 102.6 deg F, is diaphoretic, and ahs a
productive cough.  The nurse include which of the following measures in the plan of
care?
a. Position changes every 4 hours
b. sunctioning to clear secretions
c. Frequent linen changes
d. Frequent offering of bed pan
 
55. Bed rest is prescribed for a client with pneumonia during acute phase of the illness.  Bed
rest serves which of the following purposes?
a. it reduces the cellular demand for oxygen
b. it decreases the episodes of coughing
c. it promotes safety
d. it promotes clearance of secretions
 
56. The cyanosis that accompanies bacterial pneumonia is primarily caused by which of the
following?
a. decreased cardiac output
b. pleural effusion
c. inadequate peripheral circulation
d. decreased oxygenation of the blood
 
57. A client with pneumonia is experiencing pleuritic chest pain.  Which of the following
describes pleuritic chest pain?
a. a mild but constant aching in the chest
b. severe midsternal pain
c. moderate pain that worsen on inspiration
d. muscle spasm pain that accompanies coughing
 
58. Which of the following measures would most likely be successful in reducing pleuritic
cheast pain in client with pneumonia?
a. encourage the client to breath slowly
b. have the client practice abdominal breathing
c. offer the client incentive spirometry
d. Teach the client to splint the ribcage when coughing
 
59. Aspirin is administered to clients with pneumonia because of its antipyretic and
a. analgesic effects
b. anticoagulant effects
c. adrenergic effects
d. antihistamine effects
 
60. Which of the following mental status changes may occur when a client with pneumonia is
first experiencing hypoxia?
a. coma
b.  apathy
c. Irritability
d. Depression
 
61. The client with pneumonia develops mild constipation, and the nurse administers
docusate sodium (COlace) as ordered.  This drugs works by
a. softening the stool
b. lubricating the stool
c. increasing stool bulk
d. stimulating peristalsis
 
62,  A client with pneumonia has a temperature ranging between 101 and 102 deg F and periods
of diaphoresis.  Based on this information, which of the following nursing interventions would
be priority?
            A.  Maintain Complete bed rest
            b.  administer oxygen therapy
            c.  provide frequent linen changes
            d.  provide fluid intake of 3 liters/day
 
62. Which of the following would be appropriate expected outcome for an elderly client
recovering from bacterial pneumonia?
a. A respiratory rate of 25 to 30 breaths/ min
b. The ability to perform activities of daily living
c. A maximum lost of 5 to 10 pounds of body weight
d. Chest pain that is minimized by splinting the ribcage.
 
63. Which of the following symptoms is common in clients with active tuberculosis?
a. weight loss
b. increased appetite
c. dyspnea on exertion
d. mental status changes
 
64. The nurse obtains a sputum specimen from a client with suspected TB for laboratory
study.  Which of the following laboratory techniques is most commonly used to identify
tubercle bacilli in sputum.
a. Acid-fast testing
b. Sensitivity testing
c. Agglutination testing
d.  Dark field illumination
 
65. Which of the following antituberculosis drugs can cause damage to the eight cranial
nerve?
a. Streptomycin
b. INH
c. Para-aminosalisylic acid
d. Ethambutol Hcl
 
66. The nurse should teach clients that the most common route of transmitting tubercle bacilli
from person to person is through contaminated
a. dust particles
b. droplet nuclei
c. water
d. eating utensils
 
67. What is the rationale that supports multidrug treatment for clients with tuberculosis?
a. Multiple drugs potentiate the drug actions
b. Multiple drugs reduce undesireable side effects
c. Multiple drugs allow reduced dosages to be given
d. Multiple drugs reduce development of resistant strains of the bacteria
 
68. The client with TB is to be discharged home with community health nurse follow-up.  Of
the following interventions, which would have the highest priority?
a. offering the client emotional support
b. teaching the client about the disease and its treatment
c. coordinating various agency services
d. Assessing the client’s environment for sanitation
 
69. Which of the following techniques for administering Mantoux test is correct?
a. Hold the needle and syringe almost parallel to the client’s skin
b. Pinch the skin when inserting the needle
c. Aspirate before injecting the medication
d. Massage the site after injecting the medication
 
70. Which of  the following family members  exposed to TB would be at highest risk for
contracting the disease?
a. 45 year old mother
b. 17 year old daughter
c. 8 year old son
d. 76 year old grandmother
 
71. A client has a positive reaction to the Mantoux test.  The nurse correctly interprets this
reaction to mean that the client has:
a. active tuberculosis
b. had contact with M. tuberculosis
c. developed a resistance to tubercle bacilli
d. developed  passive immunity to TB
 
72. INH treatment is associated with the development of peripheral neuropathies.  Which of
the following interventions would the nurse teach the client to help prevent this
complication?
a. Adhere to a low-cholesterol diet
b. Supplement the diet with pyridoxine (Vit. B6)
c. Get extra rest
d. Avoid excessive sun exposure.
 
73. The nurse should caution sexually active female clients taking INH that the drug has
which of the following effects?
a. Increases the risk of vaginal infection
b. Has mutagenic effects on ova
c. Decreases the effectiveness of oral contraceptives
d. Inhibits ovulation
 
74. Clients who have had active TB are at risk for recurrence.  Which of the following
conditions increases that risk?
a. Cool and damp weather
b. Active exercise and exertion
c. Physical and emotional stress
d. Rest and inactivity
 
75. The nurse should include which of the following instructions when developing a teaching
plan for clients who are receiving INH and rifampicin for treatment of TB?
a. Take the medications with antacids
b. Double the dosage if a drug dose is forgotten
c. Increase intake of dairy products
d. Limit alcohol intake
 
76. During the acute stage of meningitis, a 3 year old child is restless and irritable.  Which of
the following would be most appropriate to institute?
a. limiting conversation with the child
b. keeping extraneous noise to a minimum
c. allowing the child to play in the minimum
d. Performing treatments quickly
 
77. Which of the following would lead the nurse to suspect that a child with meningitis has
developed disseminated intravascular coagulation?
a. hemorrhagic skin rash
b. edema
c. cyanosis
d. dyspnea on exertion
 
78. When interviewing the parents of a 2 year old child, a history of the following illnesses
would lead the nurse to suspect pneumococcal meningitis?
a. bladder infection
b. middle ear infection
c. fractured clavicle
d. septic arthritis
 
79. A preschooler with pneumonococci meningitis is receiving IV antibiotic therapy.  When
discontinuing the IV therapy, the nurse allows the child to apply a dressing on the area
where the needle is removed.  The rationale for doing so is based on the interpretation
that a child in this age group has need to accomplish which of the following?
a. trust those caring for her
b. find diversional activities
c. protect the image of an intact body
d. relieve the anxiety of separation from home
 
80. The physician has ordered a culture for the clients with suspected gonorrhea.  The nurse
should obtain a culture of:
a. blood
b. nasopharygeal secretions
c. stool
d. genital secretions
 
81.  A hospitalized preschooler with meningitis who is to be discharged becomes angry when the
discharged is delayed.  Which of the following play activities would most appropriate at this
time?
            a.  Reading the child a story
            b.  Painting with watercolors
            c. pounding on a pegboard
            d.  stacking a tower of blocks
 
81. Which of the following would be an important assessment finding for an 8 month of
infant admitted with severe diarrhea?
a. absent bowel sounds
b. pale yellow urine
c. normal skin elasticity
d. depressed anterior fontanel
 
82. Which of the following would be the best activity for the nurse to include in the plan of
care for an infant experiencing severe diarrhea?
a. Monitoring the total 8 hour formula intake
b. Weighing the infant each day
c. Checking the anterior fontanel every shift
d. Monitoring abdominal skin turgor every shift
 
83. The physician orders an intravenous infusion of 5 % dextrose in .25 normal saline to be
infused at 2 ml/kg/hour in an infant who weigh 9 lb.  How many ml/hr of the solution
should the nurse infuse?
 
84. Which of the following would be most appropriate for the nurse to teach the mother of a
6 month old infant hospitalized with severe diarrhea to help comfort her infant who is
fussy?
 
a. Offering a pacifier
b. Placing a mobile above the crib
c. Sitting at the crib side talking to the infant
d. Turning the television on to cartoons
 
85. which of the following nursing diagnosis would be appropriate for the nurse to identify as
a priority diagnosis for an infant just admitted to the hospital with a diagnosis of
gastroenteritis?
a. Pain related to repeated episodes of vomiting
b. Deficient Fluid volume related to excessive losses from severe diarrhea
c. Impaired parenting related to infant’s loss of fluid
d. Impaired Urinary Elimination related to increased fluid intake feeding pattern
 
86. Which of the following would the nurse use to determine achievement of the expected
outcome for an infant with severe diarrhea and nursing diagnosis of Deficient Fluid
volume related to passage of profuse amounts of watery diarrhea?
a. Moist mucous membranes
b. Passage of a soft, formed stool
c. Absence of diarrhea for a 4 hour period
d. Ability to tolerate intravenous fluids well
 
87. Which of the following would the nurse include when teaching the father of an infant just
admitted with gastroenteritis about initial treatment for his infant?
a. the infant will receive no liquids by mouth
b. IV antibiotics will be started
c. The infant will be placed in a mist tent
d. An iron fortified formula will be used
 
88. The nurse teaches the father of an infant hospitalized with gastroenteritis about the next
step of the treatment plan once the infant’s condition has been controlled.  The nurse
would determine that the father understands when he explains that which of the following
will occur with his infant?
a. The infant will receive clear liquids for a period of time
b. Formula and juice will be offered
c. Blood will be drawn daily to test for anemia
d. The infant will be allowed to go to the play room.
 
89. A child is admitted to the pediatric unit with the diagnosis of severe gastroenteritis. 
Which of the following would be most appropriate for the nurse to do?
a. Institute standard precautions
b. Place the child in a semiprivate room
c. Use regular eating utensils
d. Single- bag all linens
 
90. Which of the following would most likely alert the nurse to the possibility that a
preschooler is experiencing moderate dehydration?
a. Vomiting
b. Diaphoresis
c. Absence of tear formation
d. Decreased urine specific gravity
 
91. The physician orders IV fluid replacement therapy with potassium chloride to be added
for a child with severe gastroenteritis.  Before adding the potassium chloride to the IV,
which of the following assessments would be most important?
a. Ability to void
b. Passage of stool today
c. Baseline echocardiogram
d. Serum calcium level
 
92. Which of the following would first alert the nurse to suspect that a child with severe
gastroenteritis who has been receiving IV therapy for the past several hours may be
developing circulatory overload?
a. a drop in blood pressure
b. change to slow, deep respirations
c. auscultation of moist crackles
d. marked increase in urine output
 
93. The stool culture of a child with profuse diarrhea reveals Salmonella bacilli.  After
teaching the mother about the course of Salmonella enteritis, which of the following
statements by the mother indicates effective teaching?
a. “Some people become carriers and stay infectious for a long time”
b. “ After the acute stage passes the organism is usually not present in the stool.”
c. “ Although the organism may be alive indefinitely, in time it will be of no danger
to anyone.”
d. “ If my child continues to have the organism in the stool, an antitoxin can help
destroy the organism.”
 
94. The child is started on a soft diet after having been on clear liquids following an episode
of severe gastroenteritis.  When helping the mother choose foods for her child, which of
the following foods would be most appropriate?
a. muffins and eggs
b. bananas and rice cereal
c. bran cereal and a bagel
d. pancakes and sausage
 
95. When assessing a child diagnosed with diarrhea due to Salmonella, for which of the
following possible sources would the nurse be alert during history taking?
a. Nonrefrigerated custard
b. A pet canary
c. Undercooked eggs
d. Unwashed fruit
 
96. Interferon alfa-2b has been prescribed to treat a client with chronic hepatitis B.  What
side effect is most commonly associated with the administration of interferon alpha-2b/
a. Retinophaty
b. Constipation
c. Flu like symptoms
d. Hypoglycemia
 
97. A 19 year old male client is diagnosed with Chlamydia. Zithromax 1 gram is ordered. 
The supply of zithromax is in 250 milligram tablets.  The number of tablets to be
administered is:
 
a. 4 tablets                                               c. 6 tablets
b. 2 tablets                                               d. 8 tablets
 
98. A parent asks the nurse about headlice (pediculosis capitis) infestation during a visit to
the clinic.  Which of the following symptoms would the nurse tell the parent is most
common in a child infected with head lice?
 
a. itching of the scalp
b. scaling of the scalp
c. serous weeping of the scalp surface
d. pinpoint hemorrhagic spots on the scalp surface
 
99. After teaching the parents about the cause of ringworm of the scalp (tinea capitis), which
of the following, if stated by the father, indicates the successful teaching?
a. overexposure to the sun
b. Infestation with a mite
c. Fungal infection of the scalp
d. An allergic reaction
 
100. A mother asks the nurse, “ How did my children get pinworms?”  the nurse
explains that pinworms are most commonly spread by which of the following when
contaminated?
a. food
b. hands
c. animals
d. toilet seats
 
 
CD COMPRE EXAM 2
KEY ANSWER
 
1. A nurse is performing an assessment of a prenatal client being seen in the clinic for the
first time.  Following the assessment, the nurse determines that which piece of data places
the client into the high risk category for contracting human immunodeficiency virus
(HIV)?
a. living in the area where population rate of HIV infection is low.
b. A history of IV drug use in the past year
c. History of one sexual partner within the past 10 years
d. A spouse who is heterosexual and had only one sexual partner in the past 10
years.
 
2. A client with AIDS gets recurrent Candida infections (thrush) of the mouth.  The nurse
has given instructions to the client to minimize the occurrence of thrush and determines
that the client understands the instructions if which statement is made by the client?
a. “ I should brush my teeth and rinse my mouth once a day.”
b. “ I should use a strong mouthwash at least once a week.”
c. “Increasing red meat in my diet will keep this from recurring.”
d. “I should use warm saline or water to rinse my mouth.”
 
3. A nurse is teaching a client with AIDS how to avoid food borne illness.  The nurse
instructs the client to prevent acquiring infection from food by avoiding which of the
following items?
a. raw oysters
b. pasteurized milk
c. products with sorbitol
d. bottled water
 
4. A client with histoplasmosis has an order for ketoconazole (Nizoral).  The nurse teaches
the client to do which of the following while taking this medication?
a. Take the medication on an empty stomach
b. Take the medication with an antacid
c. Avoid exposure to sunlight
d. Limit alcohol to 2  ounces per day
 
5. A nursing instructor asks a nursing student to describe live or attenuated vaccines.  The
student tells the instructor that these types of vaccines are:
a. vaccines that have their virulence (potency) diminished so as to not produce a full-
blown clinical illness.
b.  vaccines that contain pathogens made inactive by either chemical or heat.
c.  bacterial toxins that have been made inactive by either chemicals or heat.
d.  vaccines that have been obtained from the pooled blood of many people and provide
antibodies to a variety of diseases.
 
6. A nurse is performing an assessment on a 3 year old child with chicken pox.  The child’d
mother tells the nurse that the child keeps scratching at night, and the nurse teaches the
mother about measures that will prevent an alteration in skin integrity.  Which statement
by the mother indicates that teaching was effective?
a. “ I will apply generous amounts of a cortisone cream to prevent itching.”
b. “ I need to place white gloves on my child’s hands at night.”
c. “I need to keep my child ia a warm room at night so that covers will not cause my
child to scratch.”
d. “ I will give my child a glass of warm milk at bedtime to help my child sleep.”
 
7. A nurse instructs a client with hepatitis about measures to control fatigue.  The nurse
determines that the client needs additional instructions if the client states to:
a. plan rest periods after meals
b. rest between activities
c. perform personal hygiene if not fatigue
d. complete all daily activities in the morning when the client is most rested
 
8. A clinic nurse has provided home care instructions to a female client who has been
diagnosed with recurrent trichomoniasis.  Which statement by the client indicates a need
for further instructions?
a. “ I need to perform good perineal hygiene.”
b. “ I need to refrain from sexual intercourse.”
c. “ I need to discontinue treatment if my menstrual cycle begin.”
d. “ I need to take metronidazole (Flagyl) for seven days.”
 
9. A client with AIDS has a nursing diagnosis of Imbalanced Nutrition: Less Than Body
Requirements.  The nurse has instructed the client  about methods to maintain and
increase weight.  The nurse determines that the client would benefit  from further
instruction if the client stated to:
a. Eat low-calorie snacks between meals
b. Eat small, frequent meals throughout the day
c. Consume nutrient-dense foods and beverages
d. Keep easy-to-prepare foods available in the home
 
10. A nurse is teaching a client with histoplasmosis infection about prevention of future
exposure to infectious sources.  The nurse determines that the client needs further
instructions if the client states that the potential infectious sources include:
a. grape arbors
b. mushroom cellars
c. floors of chicken houses
d. bird droppings
 
11. A clinic nurse provides instructions to a mother regarding the care of her child who is
diagnosed with croup.  Which statement by the mother indicates a need for further
instructions?
a. “ I will place a cool mist humidifier next to my child’s bed.”
b. “Sips of warm fluids during a croup attack will help.”
c. “ I will give Tynelol for the fever.”
d. “ I will give cough syrup every night at bedtime.”
 
12. A client with Chlamydia infection has received instructions on self-care and prevention
of further infection.  The nurse determines that the client needs reinforcement of the
instructions if the client states to:
a. Reduce the chance of reinfection by limiting the number of sexual partners
b. Use latex condoms to prevent disease transmission
c. Return to the clinic as requested for follow-up culture in 1 week.
d. Use antibiotics prophylactically to prevent symptoms of Chlamydia
 
 
13. A child is seen in the health care clininc and initial testing for HIV is performed because
of the child’s exposure to HIV infection.  Which home care instruction would the nurse
provide to the parents of the child?
a. Avoid all immunization until the diagnosis is established
b. Avoid sharing toothbrushes
c. Wipe up any blood spills with soap and water and allow to air dry
d. Wash hands with half-strength bleach if they come in contact with the child’s
blood
 
14. A nurse is caring for a client with AIDS.  Which finding noted in the client indicates the
presence of an opportunistic respiratory infection?
a. white plaques located on the oral mucosa
b. fever, exertional dypnea, and nonproductive cough
c. loss of sight
d. ulcerated perirectal lesions
 
15. A nurse is caring for a child diagnosed with rubeola.  The nurse notes that the physician
has documented the presence of Koplik spots.  Based on this documentation, which of the
following would the nurse expect to note on assessment of the child.
a. petechiae spots that are reddish and pinpoint on the soft palate.
b. Whitish vesicles located across the chest
c. Small, blue-white spots with a red base found on the buccal mucosa
d. Pinpoint petechiae noted on both legs
 
16. A mother brings the child to the health care clinic.  The child has been complaining of
severe headaches and has been vomiting.  The child has a high fever, and the nurse notes
the presence of nuchal rigidity in the child.  The nurse suspects a possible diagnosis of
bacterial meningitis. The nurse continues to assess the child for the presence of Kernig’s
sign.  Which finding would indicate the presence of this sign.
a. Inaability of the child to extend the legs fully when lying  supine
b. Flexion of the hips when the neck is flexed from a lying position
c. Pain when the chin is pulled down to the chest
d. Calf pain when the foot is dorsiflexed
17. A nurse determines that a Mantoux tuberculin skin test is positive.  In order to most
accurately diagnose TB, the nurse plans to consult with the physician to follow-up the
skin test with a:
a. Chest X-RAY
b. CT scan of the chest
c. Sputum culture
d. CBC
 
18. A nurse is planning care for a child with an infectious and communicable disease.  The
nurse determines that the primary goal is that the:
a. Child will experience only minor complication
b. Child will not spread the infection to others
c. Public health department will be notified
d. Child will experience mild discomfort
 
19. A mother of a child with mumps calls the health care clinic to tell the nurse that the child
has been lethargic and vomiting.  The nurse most appropriately tells the mother:
a. to continue to monitor the child
b. that lethargy and vomiting are normal manifestation of mumps
c. to bring the child to the clinic to be seen by the physician
d. that as long as there is no fever, there is nothing to be concerned about
 
20. A child with a diagnosis of hepatitis B is being cared for at home.  The mother of the
child calls the health care clinic and tells the nurse that the jaundice seems to be
worsening.  The nurse makes which response to the mother?
a. “ The hepatitis may be spreading.”
b. “ You need to bring the child to  the health care clinic to see the physician.”
c. “The jaundice may appear to get worse before it resolves.”
d. “ It is necessary to isolate the child from the others.”
 
21. A nurse is planning to teach a client who is newly diagnose with TB on how to prevent
the spread of TB.  Which instruction would be least effective in preventing the spread of
TB
a. Teach the client to cover the mouth when coughing
b. Teach the client to sterilize dishes at home
c. Teach the client to properly dispose of facial tissues
d. Teach the client that close contacts should be tested for TB
 
22. A nurse performs an initial assessment on a pregnant client and determines that the client
is at risk for toxoplasmosis.  The nurse would teach the client which of the following to
prevent exposure to this disease?
a. Wash hands only before meals
b. Eat raw meats
c. Avoid exposure to litter boxes used by cats.
d. Use tropical corticosteroids treatments prophylactically.
 
 
23. A home care nurse begins caring for a 25 year old female client who has just been
diagnosed with HIV infection.  The client asks the nurse, “ How could this have
happened?”  The nurse responds to the question based on the most frequent mode of HIV
transmission, which is
a. hugging HIV positive sexual partner without using barrier precautions.
b. Inhaling cocaine
c. Sharing food utensils with an HIV positive person without proper cleaning of the
utensils
d. Having sexual intercourse with an HIV positive person without using a condom
 
24. A client with HIV is taking zidovudine (AZT). AZT is a drug that acts to:
a. destroy the virus
b. enhance the body’s antibody production
c. slow replication of the virus
d. neutralize the toxins produced by the virus
 
25. Women who have human papillomavirus (HPV) are at risk for development of:
a. sterility
b. cervical cancer
c. uterine fibroid tissue
d. irregular menses
 
26. Which of the following nursing diagnosis would most likely be a priority for a client with
herpes genitalis ?
a. Disturbed sleep pattern
b. Imbalance Nutrition: Less than Body Requirements
c. Pain
d. Ineffective Breathing Pattern
 
27. The primary reason that a herpes simplex virus (HSV) infection is a serious concern to a
client with HIV infection is that it
a. is an acquired immunodeficiency virus –defining illness
b. is curable only after 1 year of antiviral therapy
c. leads to cervical cancer
d. causes severe electrolyte imbalances
 
28. In Educating a client about HIV, the nurses should take into account the fact that the most
effective method known to control the spread of HIV infection is:
a. premarital serological screening
b. prophylactic treatment of exposed people
c. laboratory screening of pregnant woman
d. ongoing sex education about preventive behaviors
29. A male client with HIV infection becomes depressed and tells the nurse,” I have nothing
worth living for now.” Which of the following statements would be the best response by
the nurse?
a. “ You are yopung person and have a great deal to live for.”
b. “ You should not be too depressed; we are close to finding cure for AIDS”
c. “ You are right; it is very depressing to have HIV”
d. “ Tell me more about how you are feeling about being HIV positive.”
 
30. The organism responsible for causing syphilis is classified as a:
a. virus
b. fungus
c. rickettsia
d. spirochete
31. The typical chancre of syphilis appears as
a. a grouping of small, tender pimples
b. an elevated wart
c. a painless, moist chancre
d. an itching, crusted area.
 
32. When interviewing a client with newly diagnosed syphilis, The public health nurse
should be aware that the spread of the disease ca be controlled by
a. motivating the client to undergo treatment
b. obtaining a list of the client’s sexual contacts
c. increasing the client’s knowledge of the disease
d. reassuring the client that records are confidential.
 
33. Benzathine penicillin G, 2.4 million units IM, is prescribed as treatment for an adult
client with primary syphilis.  The intramuscular injection is administered in
a. the deltoid
b. the upper outer quadrant of the buttock
c. the quadriceps lateralis of the thigh
d. the midlateral aspect of the thigh
 
34. A priority nursing diagnosis for a client with primary syphilis
a. Deficient Knowledge relataed to lack of exposure to information about mode of
transmission
b. Pain related tocutaneous skin lesion on palms and soles
c. Ineffective tissue perfusion related to bleeding chancre
d. Disturbed body image related to alopecia
 
35. An 18 year old female college student is seen at the university health center.  She
undergoes a pelvic examination and is diagnosed with gonorrhea.  Which of the
following responses by the nurse would best when the client states that she is nervous
about upcoming pelvic examination?
a. “ Can you tell me more about how you are feeling”
b. “ You’re not alone.  Most women feel uncomfortable about this examination”
c. “ Do not worry about Dr. Smith.  He is specialist in female problems.”
d. “ We’ll do everything we can to avoid embarrassing you.”
 
36. When educating a female with gonorrhea, the nurse should emphasize that for women,
gonorrhea
a. is often marked by symptoms of dysuria or vaginal bleeding
b. does not lead to serious complication
c. can be treated but not cured
d. may not cause symptoms until serious complication occurs
 
37. Which of the following groups has experienced the greatest rise in the incidence of STDs
over the past two decades
a. teenagers
b. divorced people
c. young married couples
d. older adults
 
38. A female client with gonorrhea informs the nurse that she has had sexual intercourse with
her boyfriend and asks the nurse, “  Would he have any symptoms?”  The nurse responds
that in men the symptoms of gonorrhea include
a. impotence
b. scrotal swelling
c. urinary retention
d. dysuria
 
39. The nurse assesses the mouth and oral cavity of a client with HIV because the most
common opportunistic infection initially presents as
a. HSV lesions on the lips
b. Oral candidiasis
c. CMV infection
d. Aphthae on the gingival
 
40. The nurse is planning a community education program on how to prevent the
transmission of viral hepatitis.  Which of the following types of hepatitis is considered to
be primarily a sexually transmitted disease?
a. Hepatitis A
b. Hepatitis B
c. Hepatitis C
d. Hepatitis D
 
41. The nurse would expect the client to exhibit which of the following symptoms during
icteric phase of viral hepatitis?
a. Tarry stool
b. Yellowed sclera
c. Shortness of breath
d. Light, frothy urine
 
42. The nurse plans care for the client with Hepatitis A with the understanding that the
causative virus will be excreted from the client’s body primarily through the:
a. skin
b. feces
c. urine
d. blood
 
43. The nurse is planning a staff development program for health care staff on how to care
for the clients with hepatitis A.  Which of the following precautions would the nurse
indicate as essential when caring for clients with Hepatitis A?
a. Gowning when entering aclient’s room
b. Wearing a mask when providing care
c. Assigning the client to a private room
d. Wearing gloves when giving direct care
 
44. When developing a plan of care for the client with viral hepatitis, the nurse should
incorporate nursing orders that reflect the primary treatment.  Emphasis will be on
ensuring that the client receives which of the following?
a. adequate bed rest
b. generous fluid intake
c. regular antibiotic therapy
d. daily IV theraphy
 
45. Which of the following test results would the nurse use to assess the liver function of a
client with viral hepatitis?
a. glucose tolerance
b. creatinine clearance
c. serum transaminase
d. serum electrolytes
 
46. In a client with viral hepatitis, the nurse would closely assesses for indicators of which of
the following abnormal laboratory values?
a. prolonged PT
b. decreased blood glucose level
c. elevated serum potassium
d. decreased serum calcium
 
47. Which of the follwing diets would most likely be prescribed for a client with viral
hepatitis?
a. high fat, low protein
b. high protein, low carbohydrates
c. high crbohydrates, high calorie
d. low sodium, low fat
48. The nurse develops a teaching plan for the client about how to prevent the transmission
of hepatitis A.  Which of the following discharge instructions is appropriate for the
client?
a. Spray the house to eliminate infected insects
b. Tell the family members to try to stay away from the client.
c. Tell the family members to wash their hands frequently
d. Disinfect all clothing and eating utensils
 
49. The nurse assesses that the client with hepatitis is experiencing fatigue, weakness, and a
general feeling of malaise.  The client tires rapidly during morning care .  Based  on this
information, which of the following would be an appropriate nursing diagnosis?
a. Impaired physical mobility related to malaise
b. Self-cared Deficit related to fatigue
c. Ineffective Coping related to long term illness
d. Activity Intolerance related to fatigue
 
50. A client has been admitted to the hospital with a diagnosis of hepatitis B.  the client tells
the nurse, “ Ifeel so isolated from my friends and family.  Nobody wants to be around
me.”  What would be the most appropriate nursing diagnosis for this client?
a. Anxiety related to feelings of isolation
b. Social isolation related to significant other’s fear of contracting disease
c. Powerlessness related to lack of social support
d. Low-self Esteem related to feelings of rejection.
 
51. A 79 year old female client is admitted to the hospital with the diagnosis of bacterial
pneumonia.  While obtaining the client’s health history, the nurse learns that the client
has osteoarthritis, follows a vegetarian diet, and is very concerned with cleanliness. 
Which of the following would most likely be a predisposing factor for the diagnosis of
pneumonia?
A.     Age
B.     Osteoarthritis
c.       Vegetarian diet
d.      Daily bathing
 
52. A client with bacterial pneumonia is to be started on IV antibiotics.  Which of the
following diagnostic tests must be completed before antibiotic therapy begins?
a. Urinalysis
b. Sputum culture
c. Chest radiograph
d. Red blood cell count
 
53. When caring for the client who is receiving an aninoglycoside antibiotic, the nurse
monitors which of the following laboratory values?
a. Serum sodium
b. serum potassium
c. serum creatinine
d. serum calcium
 
54. a client with pneumonia has a temperature of 102.6 deg F, is diaphoretic, and ahs a
productive cough.  The nurse include which of the following measures in the plan of
care?
a. Position changes every 4 hours
b. sunctioning to clear secretions
c. Frequent linen changes
d. Frequent offering of bed pan
 
55. Bed rest is prescribed for a client with pneumonia during acute phase of the illness.  Bed
rest serves which of the following purposes?
a. it reduces the cellular demand for oxygen
b. it decreases the episodes of coughing
c. it promotes safety
d. it promotes clearance of secretions
 
56. The cyanosis that accompanies bacterial pneumonia is primarily caused by which of the
following?
a. decreased cardiac output
b. pleural effusion
c. inadequate peripheral circulation
d. decreased oxygenation of the blood
 
57. A client with pneumonia is experiencing pleuritic chest pain.  Which of the following
describes pleuritic chest pain?
a. a mild but constant aching in the chest
b. severe midsternal pain
c. moderate pain that worsen on inspiration
d. muscle spasm pain that accompanies coughing
 
58. Which of the following measures would most likely be successful in reducing pleuritic
cheast pain in client with pneumonia?
a. encourage the client to breath slowly
b. have the client practice abdominal breathing
c. offer the client incentive spirometry
d. Teach the client to splint the ribcage when coughing
 
59. Aspirin is administered to clients with pneumonia because of its antipyretic and
a. analgesic effects
b. anticoagulant effects
c. adrenergic effects
d. antihistamine effects
 
60. Which of the following mental status changes may occur when a client with pneumonia is
first experiencing hypoxia?
a. coma
b.  apathy
c. Irritability
d. Depression
 
61. The client with pneumonia develops mild constipation, and the nurse administers
docusate sodium (COlace) as ordered.  This drugs works by
a. softening the stool
b. lubricating the stool
c. increasing stool bulk
d. stimulating peristalsis
 
62,  A client with pneumonia has a temperature ranging between 101 and 102 deg F and periods
of diaphoresis.  Based on this information, which of the following nursing interventions would
be priority?
            A.  Maintain Complete bed rest
            b.  administer oxygen therapy
            c.  provide frequent linen changes
            d.  provide fluid intake of 3 liters/day
 
62. Which of the following would be appropriate expected outcome for an elderly client
recovering from bacterial pneumonia?
a. A respiratory rate of 25 to 30 breaths/ min
b. The ability to perform activities of daily living
c. A maximum lost of 5 to 10 pounds of body weight
d. Chest pain that is minimized by splinting the ribcage.
 
63. Which of the following symptoms is common in clients with active tuberculosis?
a. weight loss
b. increased appetite
c. dyspnea on exertion
d. mental status changes
 
64. The nurse obtains a sputum specimen from a client with suspected TB for laboratory
study.  Which of the following laboratory techniques is most commonly used to identify
tubercle bacilli in sputum.
a. Acid-fast testing
b. Sensitivity testing
c. Agglutination testing
d.  Dark field illumination
 
65. Which of the following antituberculosis drugs can cause damage to the eight cranial
nerve?
a. Streptomycin
b. INH
c. Para-aminosalisylic acid
d. Ethambutol Hcl
 
66. The nurse should teach clients that the most common route of transmitting tubercle bacilli
from person to person is through contaminated
a. dust particles
b. droplet nuclei
c. water
d. eating utensils
 
67. What is the rationale that supports multidrug treatment for clients with tuberculosis?
a. Multiple drugs potentiate the drug actions
b. Multiple drugs reduce undesireable side effects
c. Multiple drugs allow reduced dosages to be given
d. Multiple drugs reduce development of resistant strains of the bacteria
 
68. The client with TB is to be discharged home with community health nurse follow-up.  Of
the following interventions, which would have the highest priority?
a. offering the client emotional support
b. teaching the client about the disease and its treatment
c. coordinating various agency services
d. Assessing the client’s environment for sanitation
 
69. Which of the following techniques for administering Mantoux test is correct?
a. Hold the needle and syringe almost parallel to the client’s skin
b. Pinch the skin when inserting the needle
c. Aspirate before injecting the medication
d. Massage the site after injecting the medication
 
70. Which of  the following family members  exposed to TB would be at highest risk for
contracting the disease?
a. 45 year old mother
b. 17 year old daughter
c. 8 year old son
d. 76 year old grandmother
 
71. A client has a positive reaction to the Mantoux test.  The nurse correctly interprets this
reaction to mean that the client has:
a. active tuberculosis
b. had contact with M. tuberculosis
c. developed a resistance to tubercle bacilli
d. developed  passive immunity to TB
 
72. INH treatment is associated with the development of peripheral neuropathies.  Which of
the following interventions would the nurse teach the client to help prevent this
complication?
a. Adhere to a low-cholesterol diet
b. Supplement the diet with pyridoxine (Vit. B6)
c. Get extra rest
d. Avoid excessive sun exposure.
 
73. The nurse should caution sexually active female clients taking INH that the drug has
which of the following effects?
a. Increases the risk of vaginal infection
b. Has mutagenic effects on ova
c. Decreases the effectiveness of oral contraceptives
d. Inhibits ovulation
 
74. Clients who have had active TB are at risk for recurrence.  Which of the following
conditions increases that risk?
a. Cool and damp weather
b. Active exercise and exertion
c. Physical and emotional stress
d. Rest and inactivity
 
75. The nurse should include which of the following instructions when developing a teaching
plan for clients who are receiving INH and rifampicin for treatment of TB?
a. Take the medications with antacids
b. Double the dosage if a drug dose is forgotten
c. Increase intake of dairy products
d. Limit alcohol intake
 
76. During the acute stage of meningitis, a 3 year old child is restless and irritable.  Which of
the following would be most appropriate to institute?
a. limiting conversation with the child
b. keeping extraneous noise to a minimum
c. allowing the child to play in the minimum
d. Performing treatments quickly
 
77. Which of the following would lead the nurse to suspect that a child with meningitis has
developed disseminated intravascular coagulation?
a. hemorrhagic skin rash
b. edema
c. cyanosis
d. dyspnea on exertion
 
78. When interviewing the parents of a 2 year old child, a history of the following illnesses
would lead the nurse to suspect pneumococcal meningitis?
a. bladder infection
b. middle ear infection
c. fractured clavicle
d. septic arthritis
 
79. A preschooler with pneumonococci meningitis is receiving IV antibiotic therapy.  When
discontinuing the IV therapy, the nurse allows the child to apply a dressing on the area
where the needle is removed.  The rationale for doing so is based on the interpretation
that a child in this age group has need to accomplish which of the following?
a. trust those caring for her
b. find diversional activities
c. protect the image of an intact body
d. relieve the anxiety of separation from home
 
80. The physician has ordered a culture for the clients with suspected gonorrhea.  The nurse
should obtain a culture of:
a. blood
b. nasopharygeal secretions
c. stool
d. genital secretions
 
81.  A hospitalized preschooler with meningitis who is to be discharged becomes angry when the
discharged is delayed.  Which of the following play activities would most appropriate at this
time?
            a.  Reading the child a story
            b.  Painting with watercolors
            c. pounding on a pegboard
            d.  stacking a tower of blocks
 
81. Which of the following would be an important assessment finding for an 8 month of
infant admitted with severe diarrhea?
a. absent bowel sounds
b. pale yellow urine
c. normal skin elasticity
d. depressed anterior fontanel
 
82. Which of the following would be the best activity for the nurse to include in the plan of
care for an infant experiencing severe diarrhea?
a. Monitoring the total 8 hour formula intake
b. Weighing the infant each day
c. Checking the anterior fontanel every shift
d. Monitoring abdominal skin turgor every shift
 
83. The physician orders an intravenous infusion of 5 % dextrose in .25 normal saline to be
infused at 2 ml/kg/hour in an infant who weigh 9 lb.  How many ml/hr of the solution
should the nurse infuse?
 
84. Which of the following would be most appropriate for the nurse to teach the mother of a
6 month old infant hospitalized with severe diarrhea to help comfort her infant who is
fussy?
 
a. Offering a pacifier
b. Placing a mobile above the crib
c. Sitting at the crib side talking to the infant
d. Turning the television on to cartoons
 
85. which of the following nursing diagnosis would be appropriate for the nurse to identify as
a priority diagnosis for an infant just admitted to the hospital with a diagnosis of
gastroenteritis?
a. Pain related to repeated episodes of vomiting
b. Deficient Fluid volume related to excessive losses from severe diarrhea
c. Impaired parenting related to infant’s loss of fluid
d. Impaired Urinary Elimination related to increased fluid intake feeding pattern
 
86. Which of the following would the nurse use to determine achievement of the expected
outcome for an infant with severe diarrhea and nursing diagnosis of Deficient Fluid
volume related to passage of profuse amounts of watery diarrhea?
a. Moist mucous membranes
b. Passage of a soft, formed stool
c. Absence of diarrhea for a 4 hour period
d. Ability to tolerate intravenous fluids well
 
87. Which of the following would the nurse include when teaching the father of an infant just
admitted with gastroenteritis about initial treatment for his infant?
a. the infant will receive no liquids by mouth
b. IV antibiotics will be started
c. The infant will be placed in a mist tent
d. An iron fortified formula will be used
 
88. The nurse teaches the father of an infant hospitalized with gastroenteritis about the next
step of the treatment plan once the infant’s condition has been controlled.  The nurse
would determine that the father understands when he explains that which of the following
will occur with his infant?
a. The infant will receive clear liquids for a period of time
b. Formula and juice will be offered
c. Blood will be drawn daily to test for anemia
d. The infant will be allowed to go to the play room.
 
89. A child is admitted to the pediatric unit with the diagnosis of severe gastroenteritis. 
Which of the following would be most appropriate for the nurse to do?
a. Institute standard precautions
b. Place the child in a semiprivate room
c. Use regular eating utensils
d. Single- bag all linens
 
90. Which of the following would most likely alert the nurse to the possibility that a
preschooler is experiencing moderate dehydration?
a. Vomiting
b. Diaphoresis
c. Absence of tear formation
d. Decreased urine specific gravity
 
91. The physician orders IV fluid replacement therapy with potassium chloride to be added
for a child with severe gastroenteritis.  Before adding the potassium chloride to the IV,
which of the following assessments would be most important?
a. Ability to void
b. Passage of stool today
c. Baseline echocardiogram
d. Serum calcium level
 
92. Which of the following would first alert the nurse to suspect that a child with severe
gastroenteritis who has been receiving IV therapy for the past several hours may be
developing circulatory overload?
a. a drop in blood pressure
b. change to slow, deep respirations
c. auscultation of moist crackles
d. marked increase in urine output
 
93. The stool culture of a child with profuse diarrhea reveals Salmonella bacilli.  After
teaching the mother about the course of Salmonella enteritis, which of the following
statements by the mother indicates effective teaching?
a. “Some people become carriers and stay infectious for a long time”
b. “ After the acute stage passes the organism is usually not present in the stool.”
c. “ Although the organism may be alive indefinitely, in time it will be of no danger
to anyone.”
d. “ If my child continues to have the organism in the stool, an antitoxin can help
destroy the organism.”
 
94. The child is started on a soft diet after having been on clear liquids following an episode
of severe gastroenteritis.  When helping the mother choose foods for her child, which of
the following foods would be most appropriate?
a. muffins and eggs
b. bananas and rice cereal
c. bran cereal and a bagel
d. pancakes and sausage
 
95. When assessing a child diagnosed with diarrhea due to Salmonella, for which of the
following possible sources would the nurse be alert during history taking?
a. Nonrefrigerated custard
b. A pet canary
c. Undercooked eggs
d. Unwashed fruit
 
96. Interferon alfa-2b has been prescribed to treat a client with chronic hepatitis B.  What
side effect is most commonly associated with the administration of interferon alpha-2b/
a. Retinophaty
b. Constipation
c. Flu like symptoms
d. Hypoglycemia
 
97. A 19 year old male client is diagnosed with Chlamydia. Zithromax 1 gram is ordered. 
The supply of zithromax is in 250 milligram tablets.  The number of tablets to be
administered is:
 
a. 4 tablets                                               c. 6 tablets
b. 2 tablets                                               d. 8 tablets
 
98. A parent asks the nurse about headlice (pediculosis capitis) infestation during a visit to
the clinic.  Which of the following symptoms would the nurse tell the parent is most
common in a child infected with head lice?
 
a. itching of the scalp
b. scaling of the scalp
c. serous weeping of the scalp surface
d. pinpoint hemorrhagic spots on the scalp surface
 
99. After teaching the parents about the cause of ringworm of the scalp (tinea capitis), which
of the following, if stated by the father, indicates the successful teaching?
a. overexposure to the sun
b. Infestation with a mite
c. Fungal infection of the scalp
d. An allergic reaction
 
100. A mother asks the nurse, “ How did my children get pinworms?”  the nurse
explains that pinworms are most commonly spread by which of the following when
contaminated?
a. food
b. hands
c. animals
d. toilet seats
 
 
 
 

Вам также может понравиться